Download as pdf or txt
Download as pdf or txt
You are on page 1of 54

GATE QUESTION PAPER 2022

General Aptitude (GA)

1 – Q. 5 carry one mark each.


Q.1 After playing _________ hours of tennis, I am feeling _________ tired to walk back.
(A) too / too (B) too / two (C) two / two (D) two / too

Ans:- (D)
Explanation:
“After playing two hours of tennis, I am feeling too tired to walk back”.
Q.2 The average of the monthly salaries of M, N and S is ₹ 4000. The average of the monthly salaries
of N, S and P is ₹ 5000. The monthly salary of P is ₹ 6000. What is the monthly salary of M as a
percentage of the monthly salary of P?
(A) 50% (B) 75% (C) 100% (D) 125%
Ans:- (A)
Solution:
This is a question that can be solved by using the formula for average, which is: average = sum /
number12.
Let x
be the monthly salary of M, y
be the monthly salary of N, and z
be the monthly salary of S. Then, we have:

Solving these equations, we get:


x+y+z=12000
y+z=9000
x=3000
Therefore, the monthly salary of M as a percentage of the monthly salary of P is:
6000x×100=60003000×100=50%
38 GATE Architecture and Planning: Comprehensive Question Bank

Q.3 A person travelled 80 km in 6 hours. If the person travelled the first part with a uniform speed of
10 kmph and the remaining part with a uniform speed of 18 kmph.
What percentage of the total distance is travelled at a uniform speed of 10 kmph?
(A) 28.25 (B) 37.25 (C) 43.75 (D) 50.00
Ans:- (C)
Solution:
Let x
be the distance travelled at a uniform speed of 10 kmph, and y
be the distance travelled at a uniform speed of 18 kmph. Then, we have:

Solving these equations, we get:


x=35
y=45
Therefore, the percentage of the total distance travelled at a uniform speed of 10 kmph is:

Q.4 Four girls P, Q, R and S are studying languages in a University. P is learning French and Dutch. Q is
learning Chinese and Japanese. R is learning Spanish and French. S is learning Dutch and Japanese.
Given that: French is easier than Dutch; Chinese is harder than Japanese; Dutch is easier than
Japanese, and Spanish is easier than French.
Based on the above information, which girl is learning the most difficult pair of languages?
(A) P (B) Q (C) R (D) S
Ans:- (B)
Explanation:
We can assign a numerical value to each language based on the given information, such as:
French = 1
Dutch = 2
Japanese = 3
Chinese = 4
Spanish = 0.5
Then, we can calculate the difficulty of each pair of languages that the girls are learning, such as:
P: French + Dutch = 1 + 2 = 3 Q: Chinese + Japanese = 4 + 3 = 7
R: Spanish + French = 0.5 + 1 = 1.5 S: Dutch + Japanese = 2 + 3 = 5
Therefore, the girl who is learning the most difficult pair of languages is Q, who is learning Chinese
and Japanese.
Hemant Vilas Parulekar 39

Q.5 A block with a trapezoidal cross-section is placed over a block with rectangular
cross section as shown above.
Which one of the following is the correct drawing of the view of the 3D object as
viewed in the direction indicated by an arrow in the above figure?

(A) (B) (C) (D)

Ans: (A)
Q. 6 – Q. 10 Carry TWO marks each.
Q.6 Humans are naturally compassionate and honest. In a study using strategically placed wallets that
appear “lost”, it was found that wallets with money are more likely to be returned than wallets
without money. Similarly, wallets that had a key and money are more likely to be returned than
wallets with the same amount of money alone. This suggests that the primary reason for this
behavior is compassion and empathy.
Which one of the following is the CORRECT logical inference based on the information in the above
passage?
(A) Wallets with a key are more likely to be returned because people do not care about money
(B) Wallets with a key are more likely to be returned because people relate to suffering of others
(C) Wallets used in experiments are more likely to be returned than wallets that are really lost
(D) Money is always more important than keys
Ans:- (B)
Explanation:
The passage states that wallets with a key are more likely to be returned than wallets with the same
amount of money alone. This suggests that people are more likely to return wallets that they believe
belong to someone else, especially if they can imagine how that person might suffer if they did not get
their wallet back.
Option (A) is incorrect because the passage does not say that people do not care about money. In fact,
the passage says that wallets with money are more likely to be returned than wallets without money,
which suggests that people do care about money.
Option (C) is incorrect because the passage does not say anything about wallets used in experiments
being more likely to be returned than wallets that are really lost.
Option (D) is incorrect because the passage does not say that money is always more important than
keys. In fact, the passage suggests that people are more likely to return wallets that they believe belong
to someone else, regardless of whether the wallet contains money or keys.
Therefore, the only logical inference that can be made from the information in the passage is that people
are more likely to return wallets that they believe belong to someone else, especially if they can imagine
how that person might suffer if they did not get their wallet back. This suggests that compassion and
empathy are the primary reasons why people return lost wallets.
40 GATE Architecture and Planning: Comprehensive Question Bank

Q.7 A rhombus is formed by joining the midpoints of the sides of a unit square. What is the diameter
of the largest circle that can be inscribed within the rhombus?
(A) 1 (B) 1 (C) √2 (D) 2√2
√2 2√2
Ans:- (A)
Solution:
As shown in the diagram drawn based on the question,
the rhombus created by connecting the midpoints of square
will also be square only.
Thus, the hypotenuse (i.e., side of the rhombus)
= x√2 (where x = Δ sides)
Thus, side of rhombus = (1/2) x √2 = 1/√2
The largest circle that can be inscribed within the rhombus
will have diameter = side of rhombus (square)
Therefore, diameter of largest circle that can be inscribed within the rhombus = 1/√2
Q.8 An equilateral triangle, a square and a circle have equal areas.
What is the ratio of the perimeters of the equilateral triangle to square to circle?
(A) 3√3 ∶ 2 ∶ √𝜋 (B)3 √3 : 2 ∶ √𝜋 (C)3 √3 : 4 ∶ 2√𝜋 (D)3 √3 : 2 ∶ 2√𝜋
Ans:- (B)
Solution:
According to the web search results, the areas of an equilateral triangle, a square, and a circle are given
by the following formulas,
Area of an equilateral triangle:

where x is the side of the triangle.


Area of a square: A=a2
where a is the side of the square.
Area of a circle: A=πr2
where r is the radius of the circle.
The perimeters of these shapes are given by the following formulas:
Perimeter of an equilateral triangle:
P=3x
Perimeter of a square: P=4a
Perimeter of a circle: P=2πr
Hemant Vilas Parulekar 41

If the areas of these shapes are equal, then we can equate the formulas and solve for the ratio of the sides:

The ratio of the perimeters is then:

Therefore, the correct answer is (B) 3√3 : 2 : √𝜋.


Q.9 Given below are three conclusions drawn based on the following three statements.
Statement 1: All teachers are professors.
Statement 2: No professor is a male.
Statement 3: Some males are engineers.
Conclusion I: No engineer is a professor.
Conclusion II: Some engineers are professors.
Conclusion III: No male is a teacher.
Which one of the following options can be logically inferred?
(A) Only conclusion III is correct
(B) Only conclusion I and conclusion II are correct
(C) Only conclusion II and conclusion III are correct
(D) Only conclusion I and conclusion III are correct
Ans: (A)
Q.10 In a 12-hour clock that runs correctly, how many times do the second, minute, and hour hands of
the clock coincide, in 12-hour duration from 3 PM in a day to 3 AM the next day?
(A) 11 (B) 12 (C) 144 (D) 2
Ans:- (A)
Explanation:
The hands of a clock coincide 11 times in every 12 hours. This is because between 11 and, they coincide
only once, i.e., at 12 o’clock. The hands overlap about every 65 minutes, not every 60 minutes. Therefore,
in a 12-hour duration from 3 PM in a day to 3 AM the next day, the hands of the clock coincide 11
times.
42 GATE Architecture and Planning: Comprehensive Question Bank

PART A: Common FOR ALL CANDIDATES


Q.11 – Q .28 (NAT/MCQ/MSQ). Each question carries one mark and negative marking of 1/3rd
mark for each wrong answer in MCQ. There is no negative marking for MSQs and NATs) (Total
marks = 18)
Q.11 The concentric circles in a sun-path diagram represent ___________.
(A) Altitude angle (B) Azimuth angle (C) Day of the year (D) Hour of the day
Ans:- (A)
Explanation: Please refer GATE-2018, Q.23 for more information).
Q.12 The operational guidelines on Credit Linked Subsidy Scheme for Economically Weaker Sections
(EWS), January 2017, by the erstwhile Ministry of Housing & Urban Poverty Alleviation,
Government of India, defines EWS households as those having an annual income up to ___________
(in Indian Rupees).
(A) 2,00,000 (B) 2,50,000 (C) 3,00,000 (D) 3,50,000
Ans:- (C)
Explanation:
The following table describes Credit Linked Subsidy Scheme (CLSS) under PMAY scheme:-
Particulars EWS LIG MIG I MIG II
Household Income Upto 3L 3-6L 6-12L 12-18L
Carpet Area in sq.m. 30 60 160 200
Interest Subsidy (% p.a.) 6.5% 4% 3%
Maximum Loan Tenure 20 years
Eligible Loan Amount (Rs.) 6,00,000/- 9,00,000/- 12,00,000/-
Discounted NPV Rate 9%
Upfront amount of subsidy (Rs.) for 20 2,67,280/- 2,35,068/- 2,30,156/-
year loan
Approx. Monthly savings @ Loan Interest 2500/- 2250/- 2,200/-
of 10%
Q.13 Which of the following is a Vector Graphics Software?
(A) Inkscape
(B) Odeon
(C) Adobe Dreamweaver
(D) DesignBuilder
Ans:- (A)
Software Use
Inkscape Vector images
Odeon Acoustical analysis
Adobe dreamweaver Web Development
Design Builder 3-D modelling
Hemant Vilas Parulekar 43

Q.14 The main cable of a suspension


bridge supports the deck with
hangars. These hangars are
equidistant along the length of
the bridge and represent a
uniformly distributed load.
Assuming the cable to be
weightless as compared to the applied loading, the best approximation of the shape that the cable takes
for this loading is a ___________.
(A) Catenary curve (B) Circular arc (C) Parabolic curve (D) Hyperbolic curve

Ans:- (C)
Explanation:
A suspension bridge carries vertical loads through curved cables in tension. These loads are transferred
both to the towers and to the anchorages, which must resist the inward pull of the cables. The suspension
bridge can be viewed as an upside-down arch in tension with only the towers in compression. The
shape of the cable depends on the distribution of the load and the boundary conditions. For a uniformly
distributed load, such as the weight of the deck and the hangars, the shape of the cable is a parabola.
This is because the horizontal component of the cable tension is constant, and the vertical component
is proportional to the sag of the cable. A parabolic curve satisfies this condition and minimizes the cable
length for a given span and sag
Q.15 Arrange the following road types in descending order of accessibility.
(P) Arterial Road (Q) Expressway (R) Collector Road (S) Local Street
(A) Q-P-R-S
(B) S-R-P-Q
(C) S-P-R-Q
(D) P-Q-S-R
Ans:- (B)
Explanation:
The roads can be classified based on their speed and accessibility. As the accessibility of a road increases,
the speed reduces. Therefore, the order of descending accessibility is as follows:
Local streets are the most accessible roads, as they provide direct access to individual properties and
buildings. They have low speed limits and traffic volumes.
Collector roads are the intermediate roads that connect local streets to arterial roads. They have
moderate speed limits and traffic volumes, and they balance the functions of mobility and accessibility.
Arterial roads are the main roads that connect different areas and regions within a city or a state. They
have high speed limits and traffic volumes, and they prioritize the function of mobility over accessibility.
Expressways are the highest level of roads that connect major cities and states across the country. They
have the highest speed limits and traffic volumes, and they have limited or no direct access to adjacent
properties. They are designed for fast and uninterrupted travel.
44 GATE Architecture and Planning: Comprehensive Question Bank

Q.16 The following two-dimensional visual composition represents ___________.

(A) Interlocking (B) Intersecting (C) Interlacing (D) Interpenetrating

Ans:- (C)
Q.17 The Golden Ratio refers to ___________.
(A) 1:√2 (B) 2: (1 + √5) (C) 1:1 (D) 16:9
Ans:- B
Explanation:
The Golden Ratio refers to a special number that is approximately equal
to 1.618 and has many interesting properties in mathematics, art, and
nature. It is also known as the divine proportion, the golden mean, or phi.
It can be expressed algebraically as (1 + √5) / 2, which is the ratio of the
longer side to the shorter side of a golden rectangle.
Q.18 Hogarth’s Line of Beauty is a ___________.
(A) Horizontal straight line (B) Zigzag line (C) Vertical straight line (D) Serpentine line
Ans:- (D)
Explanation:
Hogarth’s Line of Beauty is a term and a theory in art or aesthetics used to describe
an S-shaped curved line that appears within an object, as the boundary line of an
object, or as a virtual boundary line formed by the composition of several objects.
This theory originated with William Hogarth, an 18th-century English painter,
satirist, and writer, and is an essential part of his theory of aesthetics as described in his 1753 book
The Analysis of Beauty. According to this theory, S-shaped curved lines signify liveliness and activity
and excite the attention of the viewer, as contrasted with straight lines, parallel lines, or right-angled
intersecting lines, which signify stasis, death, or inanimate objects.
Q.19 Which of the following sites were added to Ramsar List in the year 2020?
(A) Ashtamudi Wetland (B) Asan Conservation Reserve (C) Chilika Lake (D) Lonar Lake
Ans:- (B), (D)
Explanation:
The Ramsar List is a list of wetlands of international importance maintained by the Convention on
Wetlands (Ramsar, Iran, 1971). The list aims to develop and maintain an international network of
wetlands that are important for the conservation of global biological diversity and for sustaining human
life, following sites were added to the Ramsar List in the year 2020:
-Lonar Lake in Maharashtra, India
-Sur Sarovar or Keetham Lake in Agra, Uttar Pradesh, India
-Asan Conservation Reserve in Uttarakhand, India
-Kanwar Lake or Kabal Taal in Bihar, India
-Sunderban Wetlands in Kolkata, India
Hemant Vilas Parulekar 45

Q.20 Which of the following help(s) in keeping direct solar radiation out of the building?
(A) Mashrabiya (B) Badgir (C) Malquf (D) Chajja
Ans:- (A), (D)
Explanation:
Building Mashrabiya Badgir
features
Illustrated as

Description Mashrabiya is a type of wooden screen Badgir (also spelled bâdgir or badger)
or latticework that is commonly used is a traditional Iranian architecture
in traditional Arabic architecture. It is element used for passive cooling. It is
often found on the exterior of buildings, a type of windcatcher that is used to
particularly on the upper floors, and ventilate and cool buildings by drawing
serves as a sunshade and a privacy in cool air from the outside and directing
screen. Mashrabiya consists of a series it into the building. Badgir consists of a
of wooden or stone blocks or slats that tower or shaft that is connected to the
are arranged in a decorative pattern and building and extends up through the
are spaced apart to allow air to flow roof. The top of the tower is open to
through. the air and is often shaped like a funnel
or cone to help direct the airflow. A
series of openings or vents along the
tower allow the cool air to flow into the
building..
Illustrated as Malquf Chajja
46 GATE Architecture and Planning: Comprehensive Question Bank

Building features Malquf Chajja


Description Malquf is a traditional architectural A chajja (also spelled chhajja or charja)
element found in the Middle East and is a projecting eaves or overhang that
North Africa. It is a type of dome or is found on the exterior of buildings
vaulted ceiling that is used to cover a in South Asia, particularly in India. It
space and provide structural support. is a horizontal platform or ledge that
A malquf is typically made of brick or extends outward from the wall of a
stone and is formed by stacking layers of building and is supported by brackets
arched or curved blocks on top of each or pillars. Chajjas are typically found
other to create a dome-like shape. The on the upper floors of buildings and are
blocks are held in place by gravity and used to provide shade and protection
do not require any additional support, from the sun and rain. They are also
such as beams or columns. used to create a visual break in the facade
Malqufs are often found in traditional of the building and to add interest and
Islamic architecture and are used in a character to the exterior design.
variety of buildings, including mosques, Chajjas are a common feature of
palaces, and homes. They are known for traditional Indian architecture and can
their structural strength and durability, be found on a wide range of buildings,
and are able to withstand earthquakes including homes, temples, and other
and other natural disasters. Malqufs are structures. They are often decorated
also valued for their aesthetic appeal with carved stone or wood and may be
and are often decorated with intricate painted or finished in a variety of colors.
patterns and designs. In addition to Chajjas are an important element of the
their use in architecture, malqufs have architectural style of South Asia and
also been used as burial structures in contribute to the distinctive character
some parts of the Middle East. and beauty of traditional buildings in
the region.
Q.21 As per the Handbook of Professional Documents 2015, Council of Architecture, India, architects
are liable ___________.
(A) If the building is used for any other purpose than the one for which it was designed
(B) If any unauthorised changes or illegal modifications are made by the owner(s)/occupant(s)
(C) If the client suffers damage/loss due to lack of proper professional service
(D) If the architect fails to attain the standard of care as prescribed by law
Ans:- (C), (D)
Explanation:
According to the Handbook of Professional Documents 2015, Council of Architecture, India, architects
are liable for the following:
– To perform their professional services with reasonable skill and care, and to be responsible for the
quality and accuracy of their work.
– To adhere to the standards of professional conduct and ethics prescribed by the Council of Architecture.
Hemant Vilas Parulekar 47

– To comply with the provisions of the Architects Act, 1972 and the rules and regulations made
thereunder.
– To indemnify their clients against any loss or damage caused by their negligence, breach of contract,
or violation of any law.
– To maintain professional indemnity insurance as per the scale of fees and conditions of engagement
agreed with their clients.
– To inform their clients of any conflict of interest or potential conflict of interest that may arise in the
course of their professional services.
– To respect the intellectual property rights of other architects and refrain from plagiarism or
infringement.
– To cooperate with other architects and allied professionals in the interest of the public and the
profession.
– To uphold the dignity and honour of the profession and refrain from any act or omission that may
bring disrepute to the profession.
– To contribute to the advancement of the profession and the society through research, education, and
public service.
Q.22 As per the United Nations Transforming our world: The 2030 agenda for sustainable development,
2015, which of the following Sustainable Development Goals (SDGs) directly address water related
issues?
(A) SDG-1 (B) SDG-4 (C) SDG-6 (D) SDG-14
Ans:- (C), (D)
Explanation:
The earlier adopted 8 nos. Of Millennium Development Goals (MDGs) with target year of 2015 have
been replaced by adoption of 17 nos. Sustainable Development Goals (SDGs) with target year of
2030. SDG is also known as Global Goals, and were adopted by all the United Nations member States
in September 2015 at an historic UN summit. The list of SDGs is as follows:
Goal Target
1. No Poverty End poverty in all its forms everywhere.
2. Zero Hunger End hunger, and achieve food security and improved nutrition and promote
sustainable agriculture.
3. Good Health and Ensure healthy lives and promote well-being for all at all ages.
Well-being
4. Quality education Ensure inclusive and equitable quality education and promote life-learning
opportunities for all
5. Gender Equality Achieve gender equality and empower all women an girls
6. Clean water and Ensure availability and sustainable management of water and sanitation
sanitation for all
7. Affordable and clean Ensure access to affordable, reliable, sustainable and modern energy for all
energy
8. Decent work and Promote sustained, inclusive and sustainable economic growth, full and
economic growth productive employment and decent work for all
48 GATE Architecture and Planning: Comprehensive Question Bank

9. Industry, innovation Promote inclusive and sustainable industrialization and foster innovation.
and infrastructure
10. Reduced inequality Reduce inequality within and among countries
11. Sustainable cities Make cities and human settlements inclusive, safe resilient and sustainable
and communities
12. Responsible Ensure sustainable consumption and production patterns
consumption and
production
13. Climate action Take urgent action to combat climate change and its impact
14. Life below water Conserve and sustainable use of the oceans, seas and marine resources for
sustainable development
15. Life on land Protect, restore and promote sustainable use of terrestrial ecosystems,
sustainably manage forests, combat desertification and halt and reserve land
degradation and halt biodiversity loss
16. Peace and justice Promote peaceful and inclusive societies for sustainable development,
strong institutions provide access to justice for all and build effective, accountable and inclusive
institutions at all levels.
17. Partnerships to Strengthen the means of implementation and revitalize the global partnership
achieve the goal for sustainable development
Q. 23 - Q. 28 Numerical Answer Type (NAT) (NO negative marking)
Q.23 For a masonry section, the line of action of force shifts to incorporate the effects of lateral forces
and induced moments. Consider a masonry section of width 600 mm. Assuming a zero tensile
stress capacity and a linear stress-strain response for the entire domain of loading, the minimum
value of eccentricity at which the section will crack (in mm, rounded off to one decimal place) is
__________.
Ans:- 97 - 103
Solution:
Width or thickness (t) of wall section = 600mm
Minimum value of eccentricity at which section = t/6 = 600/6 = 100mm
Q.24 The maximum and minimum indoor dry bulb temperature of a room is 38o C and 34 oC, respectively.
If the corresponding outdoor maximum and minimum dry bulb temperature are 42o C and 30o C,
respectively, then the thermal damping of the room (in percentage, rounded off to two decimal
places) is ___________.
Ans:- 66-67
Solution:
Equation for the thermal damping (D) in percentage is, D = [(T-t)/T] x 100
Where, T = Outdoor temperature range or difference (in the given problem, it is 42-30 = 120C)
t = Indoor temperature range or difference (in the given problem, it is 38-34 = 40C)
By inserting these values in the equation, D = [(12-4)/12] x 100 = 66.67
Q.25 A building site measures 96 sq.cm on a scale of 1:12500. The actual area it represents (in hectare,
in integer) is ___________.
Hemant Vilas Parulekar 49

Ans:- 150
Explanation:
Similar questions were asked earlier in (Q.45) GATE-2019, (Q.45) GATE-2015, (Q.49) 2012, (Q.35)
2011, (Q. 57) 2006, (Q.82) 2005, (q. 75) 2004, (Q.37) 2002, (Q.47) 2001.
Since profile of the site is not specified, we can assume the same
Assume, the shape of the site is rectangular when measured on map having sides as 16cm and 6cm
(Area 96 sq.cm.)
Given, scale of the map = 1:12500, I.e., 1cm on map = 12500cm on actual ground.
Thus, 16cm side on map = 16 x 125000 = 2,00,000cm = 2000m on ground
Thus, 6cm side on map = 6 x 12500 = 75,000cm = 750m on ground
Thus, actual area of the site on ground = 2000 x 750sq.m. = 15,00,000sq.m.
Thus, 1 Hectare = 10,000sq.m., So, 15,00,000 = 150 Hectare
Q.26 An off-street car parking lot contains a total of 75 bays. If the parking lot was used by 687 cars over
a period of 12 hours, the average parking turn-over of the parking lot (in vehicles per hour per bay,
rounded off to two decimal places) is ___________.
Ans:- 0.76
Solution:
Parking turn-over, is defined as the ratio of total vehicles parked in duration to the number of parking
bays available. It is expressed as number of vehicles per bay by time duration.
Given, number of bays = 75, number of vehicles = 687 and time duration = 12 hours.
Thus, Parking turn-over = (687/75)/12 = 0.76
Q.27 The hydraulic radius of the following rectangular
open drainage section (in mm, rounded off to two
decimal places) is ___________.

Ans: 136.36
Solution: Hydraulic radius (RH) = A/P
Given, A = Cross-sectional area of flow &
P = Wetted perimeter
Hence, A = 600 x 250 = 1,50,000 sq.mm. &
P = 250 + 600 +250 = 1100mm.
Thus, RH = 1,50,000/1100 = 136.36mm.
Similar question was earlier asked in (Q.52) GATE-2019 & (Q.31)
50 GATE Architecture and Planning: Comprehensive Question Bank

Q.28 A town with 0.45 million population sends its entire organic waste to a composting site on a daily
basis through a truck of 15 ton carrying capacity. Assume total waste generated per capita per day
is 0.21 kg and 40% of the total waste is organic waste.
The minimum number of weekly round trips required by the truck (in integer) will be ___________.
Ans: 21
Solution:
This is a question about the calculation of the number of truck round trips required to transport organic
waste from a town to a composting site. To answer this question, we need to follow these steps:
Step 1: Calculate the total amount of organic waste generated by the town per day. We can do this by
multiplying the population, the per capita waste generation, and the percentage of organic waste. Using
the given data, we get:
Organic waste per day=0.45×106×0.21×0.4=37,800 kg
Step 2: Calculate the number of truck trips required per day. We can do this by dividing the organic
waste per day by the truck carrying capacity. Since we need to round up the number of trips to the
nearest integer, we use the ceiling function, denoted by ⌈x⌉, which gives the smallest integer that is
greater than or equal to x. Using the given data, we get:
Truck trips per day=⌈37,800/15,000 ⌉=⌈2.52⌉=3
Step 3: Calculate the number of truck trips required per week. We can do this by multiplying the truck
trips per day by the number of days in a week, which is 7. Using the previous result, we get:
Truck trips per week=3×7=21
Q.29 – Q .49 Carry TWO marks Each
Q.29 The correct sequence of the following Construction Project Development stages, as per the National
Building Code of India 2016 is ___________.
(P) Resource Planning (Q) Project Inception
(R) Commissioning and Handing over (S) Tendering
(T) Site Survey and Soil Investigation (U) Selection of Construction Methodology
(A) P-Q-R-T-U-S (B) T-Q-R-U-S-P (C) Q-T-U-P-S-R (D) Q-T-P-S-U-R
Ans:- (C)
Explanation:
The construction project development process typically involves a number of stages, including:
Project Inception (Q): This is the first stage of the project, during which the project is initiated and the
project team is formed.
Site Survey and Soil Investigation (T): In this stage, the site is surveyed to gather information about the
soil and other site conditions that will affect the project.
Selection of Construction Methodology (U): During this stage, the project team selects the construction
methods and materials that will be used for the project.
Resource Planning (P): In this stage, the project team plans and organizes the resources needed to
complete the project, including labor, materials, and equipment.
Tendering (S): This is the process of soliciting bids from contractors to perform the work.
Commissioning and Handing over (R): The final stage of the project involves commissioning the
completed building and handing it over to the owner or operator.
Hemant Vilas Parulekar 51

Q.30 Match the aspects in Group I with the corresponding items in Group II.
(A) P-4, Q-5, R-2, S-3 Group I Group II
(B) P-5, Q-1, R-4, S-2 (P) Fire safety 1. Intruder alarm
(C) P-2, Q-4, R-5, S-1 (Q) Seismic safety 2. Zero–strength barrier
(R) Water efficiency 3. Stair lift
(D) P-2, Q-5, R-4, S-3
(S)Accessible Design 4. Aerator
5. Auxiliary damper
Ans: (D)
Explanation:
Items Intruder Alarm Zero-Strength barrier
Description An intruder alarm is a security system In a fire alarm system, a zero-strength
that is designed to detect and alert the barrier is a type of partition or separation
occupants of a building or premises that is used to divide the building or
when an unauthorized person or activity structure into fire compartments. Fire
is detected. Intruder alarms typically compartments are areas of the building
consist of sensors that are placed at that are designed to contain a fire within
strategic locations around the building or a specific area, limiting its spread and
premises, and are connected to a central allowing more time for the occupants to
control panel. When a sensor is triggered, evacuate. Zero-strength barriers are used
it sends a signal to the control panel, to create these fire compartments and
which then sounds an alarm to alert the are typically made of materials that are
occupants of the building or premises. able to withstand high temperatures for
a certain amount of time.
Items Intruder Alarm Zero-Strength barrier
illustrations

Items Stair Lift Aerator


Description A stair lift is a type of mobility device An aerator is a device that is used to
that is used to help people with mobility introduce air into a liquid or other
challenges access different levels of substance. Aerators are commonly used
a building or structure. Stair lifts are in a variety of applications, including
typically installed on the stairs of a water treatment, wastewater treatment,
building and consist of a chair or platform and aquaculture. In water treatment,
52 GATE Architecture and Planning: Comprehensive Question Bank

Items Stair Lift Aerator


Description that is mounted on a track or rail. The aerators are used to improve the quality
chair or platform is moved up and down of the water by increasing the amount
the stairs by a motor, allowing the user to of dissolved oxygen in it. This can be
easily and safely access the different levels important for maintaining the health of
of the building. Stair lifts are commonly aquatic life, such as fish, and can also help
used in homes and other residential to reduce the growth of harmful bacteria
buildings to help people with mobility and other microorganisms. Aerators can
challenges, such as seniors or individuals be used in a variety of water treatment
with disabilities, navigate stairs. They can systems, including water treatment
also be found in public buildings, such as plants, wells, and reservoirs.
schools, hospitals, and office buildings,
to provide access to people with mobility
challenges.
illustrations

Items Auxiliary Damper


Description A seismic auxiliary damper is a type of device that is used to enhance the seismic
performance of a building or structure. Seismic auxiliary dampers are typically
installed in the structure to dissipate the energy of an earthquake, reducing the
amount of force that is transmitted through the structure and minimizing damage.
There are several different types of seismic auxiliary dampers, including viscous
dampers, friction dampers, and hysteretic dampers. Viscous dampers use fluid or
gas to dissipate energy, while friction dampers use friction between two surfaces
to absorb energy. Hysteretic dampers use the energy of an earthquake to deform
a material in a controlled manner, dissipating energy as the material returns to its
original shape.
Seismic auxiliary dampers are often used in conjunction with other earthquake-
resistant features, such as base isolators, energy-dissipating structural systems, and
moment-resisting frames. They are an important part of the overall seismic design
of a building or structure, and can help to reduce the risk of damage and collapse
during an earthquake.
illustrations
Hemant Vilas Parulekar 53

Q.31 Match the States in Group I with the corresponding Vernacular Building Typologies in Group II
(A) P-4, Q-5, R-3, S-2 Group I Group II
(B) P-5, Q-1, R-2, S-4 (P) Kerala 1. Morung
(C) P-5, Q-3, R-1, S-4 (Q) Jharkhand 2. Pol
(R) Nagaland 3. Dhumkuria
(D) P-4, Q-3, R-1, S-2
(S) Gujarat 4. Nalukettu
5. Ghotul
Ans :- (D)
Explanation:
Types Morung Pol
Description Morung is a traditional architectural Gujrati Pol is a traditional style of architecture
element found in the state of found in the state of Gujarat in western
Nagaland in northeastern India. It is India. It is characterized by a distinctive style
a large, communal structure that is of building that incorporates elements of
used by the Naga people as a place traditional Indian architecture, such as ornate
for social and cultural activities. carvings, arched doorways, and elaborate
Morungs are typically found in facades.
villages and are used as a meeting Gujrati Pol buildings are typically built around
place for the community, as well a central courtyard, which is surrounded by a
as a place to store traditional Naga series of interconnected rooms and corridors.
artifacts and cultural objects. The roofs of these buildings are often made
of wood and are supported by wooden beams
or columns. The walls are often decorated
with intricate carvings and paintings, and the
buildings are finished with bright colors and
intricate designs.
illustrations
54 GATE Architecture and Planning: Comprehensive Question Bank

Types Dhumkuria Nalukettu


Description Dhumkuria is a traditional Nalukettu is a traditional style of architecture
architectural element found in the found in the state of Kerala in southern India.
state of West Bengal in eastern It is a type of house that is built around a
India. It is a type of thatched roof central courtyard and is characterized by its
that is used to cover the exterior symmetrical layout and elaborate design.
of buildings, particularly in rural Nalukettu houses are typically made of wood
areas. Dhumkuria roofs are typically and are built in the shape of a rectangle or
made of bamboo or other natural square, with a thatched roof and walls made
materials and are supported by a of brick or stone.
frame of wooden beams or poles.
They are often shaped like a dome or
a cone, and are covered with a layer
of thatch, which is made from grass,
straw, or other natural materials.
illustrations

Types Ghotul
Description Ghotul (also spelled gotul) is a traditional social and educational institution found
among the Muria people of central India. It is a communal dormitory that is used
by young Muria men and women as a place to live, study, and socialize. Ghotuls are
typically found in Muria villages and are an important part of the cultural and social
life of the Muria community.
illustrations
Hemant Vilas Parulekar 55

Q.32 Match the examples in Group I with their Corresponding typologies in Group II.
(A) P-2, Q-1, R-4, S-5 Group I Group II
(B) P-4, Q-2, R-5, S-3 (P) Navi Mumbai 1. Counter Magnet
(C) P-3, Q-1, R-2, S-5 (Q) Hissar 2. Urban Agglomeration
(R) Greater Mumbai 3. Satellite Town
(D) P-3, Q-5, R-1, S-4
(S) Delhi-Mumbai Industrial 4. University Town
Corridor
5. Investment Region
Ans:- C
Explanation:
Navi Mumbai, Satellite Town: Navi Mumbai was planned as a new city to act as a counter magnet to
Mumbai, which was facing overcrowding and congestion. It was designed to be a self-sufficient urban
settlement with its own physical and social amenities, and to absorb potential migrants and attract
some of Mumbai’s population.
Hissar is a city in the state of Haryana, which has been identified as a counter-magnet area for the
National Capital Region (NCR). A counter-magnet area is an urban center that is developed as an
alternative to the main metropolitan area, in order to reduce the pressure of migration and congestion
on the latter. Hissar was selected as a counter-magnet area because of its location, population, potential
for growth, and regional importance.
Greater Mumbai Urban Agglomeration is a term that refers to the metropolitan area of Mumbai
and its satellite towns in the northern Konkan division of Maharashtra, India. It is one of the most
populous and dense urban regions in the world, with an estimated population of over 26 million people
as of 2021. It covers an area of 6,328 square kilometres (2,443 sq mi) and consists of nine municipal
corporations and eight smaller municipal councils.
Delhi-Mumbai Industrial Corridor Investment Region is a term that refers to one of the 24 industrial
projects that are planned to be developed along the Delhi-Mumbai Industrial Corridor (DMIC), a
mega infrastructure project that aims to boost industrial development and urbanization along the route
connecting Delhi and Mumbai.
Q.33 Match the Place(s)/Event(s) in Group I with the corresponding Heritage Significance/Characteristics
in Group II.
(A) P-1, Q-4, R-3, S-2 Group I Group II
(B) P-3, Q-4, R-5, S-1 (P) Chhatrapati Shivaji 1. A long interaction between people and
(C) P-2, Q-3, R-4, S-1 Terminus, Mumbai the landscape
(Q) Kumbh Mela 2. Cultural routes
(D) P-3, Q-2, R-5, S-4
(R) Walled City of Jaipur 3. Victorian Gothic revival and traditional
Indian features
(S) Rock Shelters of 4. Intangible cultural heritage
Bhimbetka
5. Traditional human settlement, land
use reflecting an interchange of ancient
Hindu and Mughal ideas
56 GATE Architecture and Planning: Comprehensive Question Bank

Ans:- (B)
Explanation:
The Chhatrapati Shivaji Terminus, Mumbai is a railway station that exhibits a fusion of Victorian
Gothic revival and traditional Indian features. It is a symbol of the colonial and modern history of India.
The Kumbh Mela is a mass Hindu pilgrimage that occurs every 12 years at four sacred river banks. It
is considered an expression of intangible cultural heritage, as it reflects the collective faith and identity
of millions of devotees.
The Walled City of Jaipur is an example of a traditional human settlement, land use reflecting an
interchange of ancient Hindu and Mughal ideas. It was planned according to Vedic architecture and
incorporates Islamic influences in its buildings and gardens.
The Rock Shelters of Bhimbetka are a series of caves that contain some of the oldest rock paintings
in the world. They reveal a long interaction between people and the landscape, spanning from the
Paleolithic to the medieval periods.
Q.34 Match the Urban Design Concepts in Group I with their corresponding Proponents in Group II.
(A) P-1, Q-2, R-3, S-4 Group I Group II
(B) P-4, Q-1, R-2, S-3 (P) Vertical theory of Urban (1) Ian Bentley
(C) P-4, Q-3, R-5, S-1 Design
(D) P-5, Q-4, R-2, S-3 (Q) Theory of Responsive (2) Gordon Cullen
Environments
(R) Serial Vision (3) Norman Pressman
(S) Winter Urbanism (4) Ken Yeang
(5) Paul Oliver
Ans:- (B)
Explanation:
The Vertical theory of Urban Design was proposed by Ken Yeang, who advocated for high-rise, mixed-
use, and eco-friendly buildings that integrate nature and urbanism.
The Theory of Responsive Environments was developed by Ian Bentley and his colleagues, who
argued that urban design should be based on the needs and preferences of the users, and that the
physical environment should be adaptable and responsive to changing situations.
Serial Vision was a concept introduced by Gordon Cullen, who described the experience of moving
through a city as a sequence of views or images that create a sense of drama and interest.
Winter Urbanism was a term coined by Norman Pressman, who explored the challenges and
opportunities of designing urban spaces for cold climates, and suggested strategies for enhancing
comfort, accessibility, and liveability in winter cities.
Hemant Vilas Parulekar 57

Q.35 In the following sketch, P, Q, R, and S refer to elements of an urban space. Identify P, Q, R, S.

(A) P-Path, Q-Vista, R- Edge, S-Landmark


(B) P-Vista, Q-Edge, R- Landmark, S-Path
(C) P-Landmark, Q-Vista, R- Path, S-Edge
(D) P-Landmark, Q-Edge, R- Path, S-Vista
Ans:- (D)
Explanation:
A Path is a linear element that connects places and facilitates movement. It can be a street, a sidewalk,
a bridge, a canal, etc.
A Vista is a visual element that creates a sense of depth and perspective. It can be a view of a landmark,
a skyline, a natural feature, etc.
An Edge is a boundary element that defines and separates spaces. It can be a wall, a fence, a river, a
highway, etc.
A Landmark is a focal element that attracts attention and serves as a reference point. It can be a building,
a monument, a sculpture, a fountain, etc.
Q.36 As per the URDPFI Guidelines 2015, match the type of Health Care Facilities in Group I to the
corresponding population served per unit in Group II.
(A) P-1, Q-2, R-3, S-4 Group I Group II
(B) P-3, Q-1, R-5, S-4 (P) Multi-Speciality Hospital (1) 15,000
(C) P-4, Q-3, R-5, S-2 (Q) Dispensary (2) 50,000
(R) Veterinary Hospital (3) 1,00,000
(D) P-5, Q-1, R-2, S-3
(S) General Hospital (4) 2,50,000
(5) 5,00,000
58 GATE Architecture and Planning: Comprehensive Question Bank

Ans:- (B)
Explanation:
Sr.no Type Area required
1 Dispensary 1 for every 15000 population 0.08 ha to 0.12
2 Nursing home, child welfare snf maternity center( 1 for every 45000 0.20 ha to 0.30 ha
to 1,00,000 population) capacity of 25 to 30 beds
3 Poly clinic with some observation beds (1 for every 1,00,000 0.20 ha to 0.30 ha
population)
4 Intermediate hospital (1 for every 1,00,000 population) capacity of 1.00 ha
80 beds
5 Intermediate hospital (1 for every 1,00,000 population) capacity of 3.70ha
200 beds
6 General hospital (1 for every 250,000 population) capacity of 300 6.00 ha
beds
7 Multi-speciality hospital (1 for 1,00.000 population) capacity of 200 9.00 ha
beds+ residentials
8 Specialty hospital (1 for every 1,00,000 population) capacity of 200 3.70 ha
beds + residentials.
Q.37 Match the plan forms in Group I with their corresponding project names in Group II.
Group I Group II
(P) (1) New Parliament of Egypt,
Cairo

(Q) (2) Apple Park Campus,


California

(R) (3) Commerzbank, Frankfurt

(S) (4) 30 St. Mary Axe, London

(5) Parliament Building, Dhaka


(A) P-3, Q-5, R-4, S-2 (B) P-4, Q-2, R-1, S-5 (C) P-1, Q-2, R-3, S-4 (D) P-3, Q-5, R-1, S-2
Hemant Vilas Parulekar 59

Ans: (D)
Explanation:
Project name Commerzbank, Frankfurt Parliament Building, Dhaka
Description The plan form of the Commerzbank is a The plan form of the Parliament Building,
triangular shape, with rounded corners Dhaka, is a triangular shape, with a
and slightly convex sides. The floors central dome and three rounded corners.
of the building and the cores at the The dome houses the main assembly hall,
three corners are organized around a which can seat 354 members. The three
central atrium, which is also triangular corners contain the offices of the president,
in shape. Each floor has three wings, the prime minister, and the speaker. The
two of which serve as office space; the building also has nine blocks, each with
third forms part of one of the nine sky four stories, that surround the dome and
gardens that are distributed throughout the corners. The blocks contain various
the building. facilities, such as committee rooms,
libraries, lounges, and dining halls.
Plan form

Project name New Parliament of Egypt, Apple Park Campus,


Cairo California
Explanation The plan form of the New Parliament of The plan form of the Apple Park Campus
Egypt, Cairo, is a circular shape, with a is a ring shape, with a large courtyard
central dome and four minarets at the in the center and a curved facade. It is
corners. The dome houses the main a symmetrical and regular plan, which
assembly hall, which can accommodate suggests a modern and efficient building
1,000 members. The minarets serve type. The building has four stories above
as observation towers and symbols of the ground and two basement levels, and
Islamic culture. The building also has covers an area of 2.8 million square feet. It
four wings that radiate from the dome, can accommodate 12,000 employees, who
containing the offices of the president, work in open-plan spaces that encourage
the prime minister, the speaker, and the collaboration and creativity.
parliamentary committees.
Plan form
60 GATE Architecture and Planning: Comprehensive Question Bank

Q.38 Match the Biosphere reserves in India in Group I with their corresponding locations in Group II.
(A) P-2, Q-1, R-4, S-3 Group I Group II
(B) P-2, Q-3, R-1, S-5 (P) Agasthyamala Biosphere (1) Western Himalayan region,
(C) P-3, Q-1, R-4, S-5 Reserve Himachal Pradesh
(Q) Nokrek Biosphere Reserve (2) Western Ghats, Kerala and
(D) P-4, Q-5, R-1, S-3
Tamil Nadu
(R) Cold desert Biosphere (3) Tura range, Meghalaya
Reserve
(S) Simlipal Biosphere Reserve (4)Kachchh,Rajkot,Surendranaga,
and Patan districts, Gujarat
(5) Mayurbhanj district, Odisha
Ans:- (B)
Explanation:
Biosphere reserves are areas of terrestrial and coastal or marine ecosystems that are recognized for
their biodiversity and cultural values, and are managed for conservation and sustainable development1.
India has 18 biosphere reserves, of which 12 are part of the World Network of Biosphere Reserves under
the UNESCO Man and the Biosphere Programme.
The correct match of the biosphere reserves in Group I with their locations in Group II is as follows:
P: Agasthyamala Biosphere Reserve is located in the Western Ghats region, covering parts of Kerala
and Tamil Nadu. It is home to a rich variety of flora and fauna, including endemic and endangered
species such as the lion-tailed macaque, the Nilgiri tahr, and the Malabar large-spotted civet. Therefore,
P-2 is correct.
Q: Nokrek Biosphere Reserve is located in the Tura range of the Meghalaya plateau, encompassing
the Nokrek National Park and several wildlife sanctuaries. It is a hotspot of biodiversity, with rare and
endemic species such as the red panda, the Asian elephant, and the citrus indica, the most primitive
ancestor of all citrus fruits. Therefore, Q-3 is correct.
R: Cold Desert Biosphere Reserve is located in the Western Himalayan region, covering parts
of Himachal Pradesh and Jammu and Kashmir. It comprises the Pin Valley National Park and the
Chandratal and Sarchu Wildlife Sanctuaries. It is characterized by high-altitude cold deserts, alpine
meadows, and snow-capped mountains, and harbors species such as the snow leopard, the Himalayan
brown bear. Therefore, R-1 is correct.
S: Simlipal Biosphere Reserve is located in the Mayurbhanj district of Odisha, covering the Simlipal
National Park and the adjoining reserve forests. It is part of the Deccan Peninsula biogeographic zone,
and features tropical moist deciduous forests, grasslands, and waterfalls. It is home to a diverse range
of wildlife, including the royal Bengal tiger, the Asian elephant, and the gaur. Therefore, S-5 is correct.
Q.39 In traditional Persian context, qanat system refers to
(A) An underground water-way, tunnelled and channelled
(B) A system where water is raised by a series of scoops fixed to a moving belt stretched between two
wheels
(C) A method of conducting water from a source-well rather than raising it
(D) A system where water is conducted from enclosure to enclosure by straightforward gravity fall
Hemant Vilas Parulekar 61

Ans:- A, C, D
Explanation:
A qanat system is a system for transporting water from an aquifer or water well to the surface, through
an underground aqueduct; the system originated approximately 3,000 years ago in what is now Iran1.
Qanats are used for irrigation and drinking water in arid regions, and are considered as an exceptional
testimony to the cultural traditions and civilizations in desert areas. Qanats have different names in
different regions, such as karez, foggara, khettara, falaj, etc.
Q.40 Which of the following is/are classified as the Principles of Universal Design?
(A) Perceptible Information
(B) Tolerance for Error
(C) Occult Balance
(D) Simple and Intuitive Use
Ans:A, B, D
Explanation:
The Principles of Universal Design are a set of guidelines for designing products, services, and
environments that are accessible and usable by all people, regardless of their abilities, age, gender, or
background. They were developed in 1997 by a group of architects, product designers, engineers, and
researchers at the Center for Universal Design at North Carolina State University.
The Principles of Universal Design are as follows:
Principle 1: Equitable Use - The design is useful and marketable to people with diverse abilities.
Principle 2: Flexibility in Use - The design accommodates a wide range of individual preferences and
abilities.
Principle 3: Simple and Intuitive Use - The design is easy to understand, regardless of the user’s
experience, knowledge, language skills, or current concentration level.
Principle 4: Perceptible Information - The design communicates necessary information effectively to
the user, regardless of ambient conditions or the user’s sensory abilities.
Principle 5: Tolerance for Error - The design minimizes hazards and the adverse consequences of
accidental or unintended actions.
Principle 6: Low Physical Effort - The design can be used efficiently and comfortably and with a
minimum of fatigue.
Principle 7: Size and Space for Approach and Use - The design provides appropriate size and space for
approach, reach, manipulation, and use regardless of the user’s body size, posture, or mobility.
Q.41 As per the URDPFI Guidelines 2015, which of the following Organoleptic and Physical parameters
comply with the acceptable limit requirements of drinking water quality?
(A) Colour: Maximum 5 Hazen units
(B) Turbidity: Maximum 1 NTU
(C) pH Value: Minimum 10
(D) Total Dissolved Solids: Maximum 500 mg/l
62 GATE Architecture and Planning: Comprehensive Question Bank

Ans:- A, B, D
Explanation:
The URDPFI Guidelines 2015 are the Urban and Regional Development Plans Formulation and
Implementation Guidelines issued by the Ministry of Housing and Urban Affairs of India. They provide
a comprehensive framework for planning and development of urban and regional areas in India.
According to the URDPFI Guidelines 2015, the acceptable limit requirements of drinking water quality
are based on the World Health Organization (WHO) Guidelines for Drinking-water Quality2. The
WHO Guidelines for Drinking-water Quality are the international reference point for the setting of
national regulations and standards for water safety in support of public health.
The WHO Guidelines for Drinking-water Quality specify the following organoleptic and physical
parameters for drinking water quality.
Colour: The colour of drinking water should be less than or equal to 15 Hazen units. Colour is an
indicator of the presence of organic matter, metals, or other substances in water. High colour may affect
the acceptability of water by consumers and interfere with disinfection processes.
Turbidity: The turbidity of drinking water should be less than or equal to 5 NTU. Turbidity is a measure
of the cloudiness of water caused by suspended particles. High turbidity may reduce the effectiveness
of disinfection, increase the risk of microbial contamination, and affect the aesthetic quality of water.
pH Value: The pH value of drinking water should be between 6.5 and 8.5. pH is a measure of the acidity
or alkalinity of water. Extreme pH values may cause corrosion or scaling of pipes and fittings, affect the
taste of water, and alter the effectiveness of disinfection.
Total Dissolved Solids: The total dissolved solids (TDS) of drinking water should be less than or equal
to 1000 mg/l. TDS is a measure of the total amount of dissolved minerals, salts, metals, and organic
matter in water. High TDS may affect the taste, odour, and appearance of water, and may also have
health implications for some consumers.
Q.42 In an ideal air-conditioning cycle shown below, which of the following statement(s) is/are true in
the segments P, Q, R, S?

(A) P: Vapour at low pressure (B) Q: Vapour at low pressure


(C) R: Liquid at high pressure (D) S: Liquid-Vapour mixture at low pressure
Hemant Vilas Parulekar 63

Ans:- A,C,D
Explanation:
The ideal air-conditioning cycle is also known as the vapor-compression refrigeration cycle. It consists
of four main components: the compressor, the condenser, the expansion device, and the evaporator.
The refrigerant, a fluid that can change its state from liquid to vapor and vice versa, flows through these
components and undergoes various thermodynamic processes.
P: This is the saturated vapor state of the refrigerant at low pressure and low temperature. It enters the
compressor, where it is compressed to a high pressure and high temperature superheated vapor state.
Therefore, option (A) is true.
Q: This is the superheated vapor state of the refrigerant at high pressure and high temperature. It leaves
the compressor and enters the condenser, where it is cooled and condensed to a high pressure and
moderate temperature liquid state. Therefore, option (B) is false.
R: This is the liquid state of the refrigerant at high pressure and moderate temperature. It leaves the
condenser and enters the expansion device, where it is throttled to a low pressure and low temperature
liquid-vapor mixture state. Therefore, option (C) is true.
S: This is the liquid-vapor mixture state of the refrigerant at low pressure and low temperature. It leaves
the expansion device and enters the evaporator, where it is heated and evaporated to a low pressure and
low temperature saturated vapor state. Therefore, option (D) is true.

Q.43 Which of the following is/are the characteristic(s) of a Mughal Garden?


(A) Symmetrical and geometrical
(B) Fountain and channelled water
(C) Winding road and untrimmed vegetation
(D) Vista with terminal building
Ans:- A, B, D
Explanation:
Refer GATE 2021 Q.30 for more information.
64 GATE Architecture and Planning: Comprehensive Question Bank

Q.44 As per the Central Pollution Control Board’s National Air Quality Index (AQI) of India 2014,
which of the following statement(s) is/are true?
(A) AQI is computed considering 8-hourly value of CO
(B) AQI is computed considering 2-hourly value of PM2.5
(C) AQI considers the O3 concentrations
(D) AQI considers the CO2 concentrations
Ans:- A, C
Explanation:
The National Air Quality Index (AQI) of India is a tool to communicate the status of air quality and its
associated health impacts to the public. It is computed based on the concentrations of eight pollutants:
PM10, PM2.5, NO2, SO2, CO, O3, NH3, and Pb.
The AQI is computed considering the 8-hourly value of CO, which is a colorless, odorless, and toxic gas
that can affect the oxygen-carrying capacity of the blood and cause various health problems2. The AQI
is also computed considering the 8-hourly value of O3, which is a gas that can irritate the respiratory
system and cause breathing difficulties, coughing, and chest pain.
The AQI is not computed considering the 2-hourly value of PM2.5, but the 24-hourly value of PM2.5,
which are fine particles that can penetrate deep into the lungs and cause respiratory and cardiovascular
diseases4. The AQI also does not consider the CO2 concentrations, as CO2 is not one of the eight
pollutants that are used to calculate the AQI.
Q.45 The decadal population data of a city are given in the following Table. The domestic water
consumption of the city is estimated to be 175 litres per capita per day in the year 2041. Considering
2011 population as the base year and using arithmetic growth method of population forecasting,
the daily domestic water demand of the city in the year 2041 (in million litres per day, rounded off
to two decimal places) will be ___________.
Year 1981 1991 2001 2011
Population 1,80,750 1,95,850 2,15,300 2,45,450
Ans:- 53.00 to 56.00
Solution:
The daily domestic water demand of the city in the year 2041 will be 5.01 million litres per day. Here is
how to find the answer:
The arithmetic growth method is a method of population forecasting that assumes that the population
grows at a constant rate over time. It estimates the future population by adding a fixed number of
individuals to the current population in each period.
To use the arithmetic growth method, we need to calculate the average rate of increase of population
(x̄) and the number of decades (n) between the base year and the future year. Then, we can apply the
formula:
Pn=P0+nx
Hemant Vilas Parulekar 65

here P_n is the future population, P_0 is the base population, n is the number of decades, and x� is the
average rate of increase of population.
Given that the base year is 2011 and the future year is 2041, we can calculate the number of decades as
follows:

Given that the population data of the city are given in the table, we can calculate the average rate of
increase of population as follows:

x =21533.33
Therefore, the future population in 2041 is:
Pn=P0+nx
P2041=P2011+3×21533.33
P2041=245450+64599.99
P2041=309949.99
To round off the future population to the nearest integer, we need to look at the decimal part and see if it
is 0.5 or more. If it is, we round up the integer part by one. If it is less than 0.5, we leave the integer part
as it is. In this case, the decimal part is 0.99, which is more than 0.5, so we round up the integer part by
one. Therefore, the future population rounded off to the nearest integer is:
P2041=309950
Given that the domestic water consumption of the city is estimated to be 175 litres per capita per day in
the year 2041, we can calculate the daily domestic water demand of the city as follows:
Water demand=Population× Consumption
Water demand=309950×175
Water demand=54241250
To convert the water demand from litres per day to million litres per day, we need to divide it by
1000000. Therefore, the water demand in million litres per day is:
66 GATE Architecture and Planning: Comprehensive Question Bank

Water demand=54.24125
To round off the water demand to two decimal places, we need to look at the third decimal place and
see if it is 5 or more. If it is, we round up the second decimal place by one. If it is less than 5, we leave the
second decimal place as it is. In this case, the third decimal place is 1, which is less than 5, so we leave
the second decimal place as it is. Therefore, the water demand rounded off to two decimal places is:
Water demand=54.24
Q.46 The activity duration, early start, early finish, late start,
and late finish (in weeks) for activities P, Q, R, and S are
shown in the following figure. The interfering float of
activity R (in weeks, in integer) is ___________.

Ans:- 6
Explanation:
Term Interfering float Free float
Definition Interfering float is the amount of time Free float is the amount of time an
an activity can be delayed or extended activity can be delayed or extended
without: without delaying the early start of its
Delaying the early start of any of its immediate successor activity.It represents
successors. Being affected by delays in its the flexibility you have to adjust a task’s
predecessors. It represents the flexibility timing without negatively impacting the
you have to adjust an activity’s timing start dates of tasks that directly depend
within a specific window, independent on it.
of both its preceding and succeeding
tasks
Formula Interfering Float (IF) = Minimum(Late Free Float (FF) = Early Start of the
Finish of Successors) - Early Finish (EF) Successor (ES(succ)) - Early Finish (EF)

Term Independent float Total float


Definition Independent float is the amount of time Total float is the maximum amount of time
an activity can be delayed or extended an activity can be delayed or extended
without delaying the early start of any of from its early start date without delaying
its successors and without being affected the project finish date.It represents the
by the delays of its predecessors. flexibility you have within a project
It represents the flexibility you have to schedule to adjust task timings without
adjust a task’s timing within a specific impacting the overall timeline.
window, independent of both its
predecessors and successors.
Hemant Vilas Parulekar 67

Formula Independent Float (IF) = Minimum(Late There are two common formulas to
Finish of Successors) - Early Finish (EF) calculate total float:
Total Float (TF) = Late Finish (LF) - Early
Finish (EF)
Total Float (TF) = Late Start (LS) - Early
Start (ES)
From the figure,
Activity Duration Predecessors Early Early Late Late Total Free
Activity intF
start finish start finish float float
P 2 -
P 10 12 10 12 0 0 0
Q 9 P Q 12 21 12 21 0 0 0
R 3 P R 12 15 24 27 12 6 6
S 2 Q,R S 21 23 27 29 6 - -
Q.47 A 230 mm thick brick wall of 10 m length and 3 m height is built using a Flemish bond. The size of
the bricks used is 230 mm x 112.5 mm x 70 mm. Assuming a mortar thickness of 5 mm, the number
of bricks required (in integer) is ___________.
Ans:- 3400-3500
Solution:
Total length of brick wall to be built=10m
As per the size of brick used, 1 unit of 3 bricks (header and stretcher as shown in the figure below) 5
mm mortar will cover length of 347.5 mm= 0.347 m
Thus, number of units ( 3 bricks each) required in single layer for covering 10 mm length=
10/0.347=28.81=29
Thus, actual number of bricks required in single layer=29×3=87…….(1)
The height of each brick is 70 mm
Thus, a single layer including mortar thickness of 5mm covers height of 75mm 0.075 m
Thus the number of courses of bricks required to built wall of height 3m=40……(2)
Thus from one and 2 the number of bricks required to build the wall of given dimension=87×40=3480
68 GATE Architecture and Planning: Comprehensive Question Bank

Q.48 The reflected ceiling plan and section of a reinforced cement concrete roof are shown in the following
Figure. All the beams are 300 mm wide, 600 mm deep (including 150 mm slab) equidistantly placed
center to center. Assuming 1% of concrete volume is occupied by reinforcement bars, the volume of
concrete (in cubic meters, rounded off to two decimal places) is ___________.

Ans:- 24.50-25.30
Solution:
The volume of roof without considering the intermediate sunks =10×10×600mm=60m3
From, the figure, the dimensions of square sunks between beams can be calculated as 2.94m×2.94m=8.64m2
Net depth of square sunk=depth of beam-dept of slab=600-150=450mm=0.45m
The volume of 9 no. of sunks=9×8.64×0.450=34.992 m3
Thus, volume of formwork available for pouring RCC (slab+beam) =60-35=25m3
It is given that; 1% volume is occupied by reinforcement bars
Thus, volume of concrete=25-(0.01×25) =24.75m3
Q.49 The following graph represents the income
distribution among the population of a country.
The Gini Coefficient of the country (rounded off
to three decimal places) is ___________.
Hemant Vilas Parulekar 69

Ans:- 0.240 to 0.27


Solution:
The Gini Coefficient is a measure of income inequality that ranges from 0 (perfect equality) to 1 (perfect
inequality). It is calculated as the ratio of the area between the line of perfect equality and the Lorenz
curve (the blue curve in the graph) to the total area under the line of perfect equality.
Gini coefficient= A/A+B
Where A= area above Lorenz curve
B= area below Lorenz curve
Here area above Lorenz curve A
= A(ΔPTS)-Area below lorez curve
=1/2×100×100=5000
Area below Lorenz curve B= A(ΔPVQ)+ A(QVUR)+ A(ΔRUTS)
=270+1260+2175=3705
Thus, Area above lorenz curve A= 5000-3705=1295
Thus,Gini coefficient= 1295/ (1295+3705)= 0.259
PART B1: FOR Architecture CANDIDATES ONLY
Q.50 – Q.56 Carry ONE mark Each
Q.50 Which of the following processes is used for surface treatment of metals?
(A) Soldering (B) Thermoplating (C) Extrusion (D) Riveting
Ans:- (B)
Explanation:
Surface treatment is a process of applying a coating or modifying the surface of a material to improve
its appearance, performance, or durability.
Process Soldering Thermoplating
Description Soldering is a process of joining Thermoplating is a type of surface
two metal pieces by melting a filler treatment that involves heating a metal and
metal (solder) between them. It is dipping it into a molten metal bath to form
not a surface treatment, but a joining a protective layer on the surface. It is also
technique. known as hot-dip galvanizing or hot-dip
coating.
Image
70 GATE Architecture and Planning: Comprehensive Question Bank

Process Extrusion Riveting


Description Extrusion is a process of forcing a Riveting is a process of fastening two metal
metal through a die to create a desired pieces by inserting and deforming a metal
shape or profile. It is not a surface pin (rivet) through holes in them. It is not a
treatment, but a forming technique. surface treatment, but a fastening technique.
Image

Q.51 Among the following monuments of ancient Greece, the only Octastyle Peripteral temple with
eight towering Doric columns lining both east and west facades is ___________.
(A) Temple of Athena (B) Temple of Apollo (C) The Parthenon (D) Temple of Horus
Ans:- (C)
Explanation:
An octastyle peripteral temple is a temple that has eight columns on the front and back facades and a
single row of columns around the main chamber (cella). A Doric column is a type of column that has a
simple capital (top part) and no base (bottom part).
The Parthenon is a famous octastyle peripteral temple that was dedicated to the goddess Athena. It was
built in the 5th century BC on the Acropolis of Athens. It is considered one of the finest examples of
classical Greek architecture.
The Temple of Athena is not a specific temple, but a general name for any temple that was dedicated
to the goddess Athena. There were many temples of Athena in ancient Greece, but not all of them were
octastyle peripteral or had Doric columns.
The Temple of Apollo is also not a specific temple, but a general name for any temple that was dedicated
to the god Apollo. There were many temples of Apollo in ancient Greece, but not all of them were
octastyle peripteral or had Doric columns.
The Temple of Horus is not a Greek temple, but an Egyptian temple that was dedicated to the god
Horus. It was built in the 2nd century BC in Edfu, Egypt. It has a different architectural style than the
Greek temples, with pylons, hypostyle halls, and hieroglyphs.
Q.52 An Ultrasonic Pulse Velocity (UPV) test was done on a hardened concrete element using a direct
transmission method as per IS 516 (Part 5/Section 1): 2018. The distance between the transducer
and receiver was 600 mm. The time taken for the induced wave to travel this distance is measured
as 0.18 milliseconds. Based on the following Table, the concrete quality grading is ___________.
Velocity (km/s) – cross probing Concrete quality grading
Above 4.4 Excellent
3.75 – 4.4 Good
3.0 – 3.75 Doubtful
Less than 3.0 Poor
(A) Excellent (B) Good (C) Doubtful (D) Poor
Hemant Vilas Parulekar 71

Ans:- (C)
Explanation:
To find the concrete quality grading, we need to calculate the ultrasonic pulse velocity (UPV) of the
concrete element. The UPV is the ratio of the distance between the transducer and receiver to the time
taken for the induced wave to travel this distance.
Given that the distance between the transducer and receiver is 600 mm and the time taken for the
induced wave to travel this distance is 0.18 milliseconds, we can calculate the UPV as follows:
UPV = Distance / Time UPV = 600 mm / 0.18 ms UPV = 3.33 m/s
To convert the UPV from m/s to km/s, we need to multiply it by 1000. Therefore, the UPV in
km/s is:
UPV = 3.33 m/s * 1000 UPV = 3.33 km/s the range in table shows doubtfull.
Q.53 Which of the following is/are example(s) of Tomb Architecture of Ancient Egypt?
(A) Step Pyramid of Zoser, Sakkara (B) Great Temple of Abu-Simbel
(C) Temple of Khons, Karnak (D) Mastabas of Gizeh
Ans:- A,D
Explanation:
The Step Pyramid of Zoser is the oldest known pyramid in Egypt, built for the pharaoh Djoser in the
27th century BCE. It consists of six mastabas (rectangular structures) of decreasing size stacked on top
of each other. It is located in the necropolis of Sakkara, near Memphis.
The Mastabas of Gizeh are the tombs of the nobles and officials who served the pharaohs of the Fourth
Dynasty (c. 2575–c. 2465 BCE). They are rectangular structures with sloping sides and flat roofs, built
of mud bricks or stone. They are located near the famous pyramids of Gizeh, such as the Great Pyramid
of Khufu.
he Great Temple of Abu-Simbel is not a tomb, but a temple dedicated to the pharaoh Ramesses II and
the gods Amun, Ra-Horakhty, and Ptah. It was built in the 13th century BCE and features four colossal
statues of Ramesses II at the entrance. It is located near the border with Sudan.
The Temple of Khons is not a tomb, but a temple dedicated to the god Khons, the son of Amun and
Mut. It was built in the 12th century BCE by the pharaoh Ramesses III and later expanded by other
rulers. It is located within the complex of Karnak, near Luxor.
Q.54 If Aluminium: Anodisation : : Glazing : X, which of the following choices represent X?
(A) Hard coating (B) External cement plastering
(C) Tempering (D) Free-standing vertical greening
Ans:- A, C
Explanation:
Anodisation is a process of forming a protective oxide layer on the surface of aluminium by electrolysis.
It increases the corrosion resistance, hardness, and durability of aluminium1.
Glazing is a process of applying a thin layer of glass or ceramic on the surface of pottery or metal by
heating. It improves the appearance, water resistance, and strength of the material2.
72 GATE Architecture and Planning: Comprehensive Question Bank

Tempering is a process of heating and cooling glass or metal to a specific temperature and rate. It
reduces the brittleness and increases the toughness of the material.
External cement plastering is a process of applying a mixture of cement, sand, and water on the
external walls of a building. It provides a smooth and protective surface for the walls. It is not analogous
to anodisation or glazing.
Free-standing vertical greening is a system of growing plants on a vertical structure that is independent
of a building.
Q.55 A blackbody radiant heating panel of 5 m2 surface area at 35 oC surface temperature is located 1 m
away from a 1 m2 surface at 20 oC. The Stefan-Boltzmann constant is 5.6703×10−8 W m−2 K−4. The
rate of radiant heat emission by the radiant heating panel (in W, rounded off to two decimal places)
is ___________.
Ans:- 2550 to 2560
Solution:
To find the rate of radiant heat emission by the radiant heating panel, we can use the Stefan-Boltzmann
law, which states that the power emitted per unit area of a blackbody is proportional to the fourth power
of its absolute temperature. Therefore, we can write:
P=σAT4
Where P is the power emitted, σ is the Stefan-Boltzmann constant, A is the surface area, and T is the
absolute temperature. We can convert the given temperatures from degrees Celsius to kelvins by adding
273.15. Thus, we have:
273.15+35=308.15k
Putting values in the equation:
P=(5.6703×10−8)(5)(308.15)4=2556.37W
Therefore, the rate of radiant heat emission by the radiant heating panel (in W, rounded off to two
decimal places) is 2560W.
Q.56 A hypothetical truss comprising of
weightless members is shown in the
following Figure. Assuming tension to be
positive and compression to be negative,
the value of force in member TU (in kN,
rounded off to one decimal place) is
___________.

Ans:- 0
Explanation:
The joint T has two collinear members (VT and TS) and third member TU is non-colinear member.
AS, there is no external force acting at joint T, thus, the non-colinear member TU is zero-force member.
Hemant Vilas Parulekar 73

Q.57 – Q .65 Carry TWO marks Each


Q.57 Match the illustrations of Arch Types in Group I with their corresponding names in Group II.
Group I Group II
(P) (1) Venetian Arch

(Q) (2) Ogee Arch

(R) (3) Moorish Multifoil Arch

(S) (4) Corbelled Arch

(5) Shouldered Arch


(A) P-2, Q-3, R-1, S-4 (B) P-3, Q-1, R-2, S-5 (C) P-3, Q-2, R-5, S-4 (D) P-5, Q-4, R-3, S-1
Ans:- (B)
Explanation:
Arch type Venetian Arch Ogee Arch
Description A Venetian arch is an arch that has An ogee arch is an arch that has a pointed
a pointed apex and two symmetrical apex and two symmetrical curves that
curves that form a flattened S-shape. It form a sharp S-shape. It is often used in
is commonly found in Venetian Gothic Gothic architecture, especially in windows
architecture, especially in palaces and and doorways.
churches.
Image
74 GATE Architecture and Planning: Comprehensive Question Bank

Arch type Moorish Multifoil Arch Corbelled Arch


Description A Moorish multifoil arch is an arch that A corbelled arch is an arch-like structure
has multiple lobes or foils on its inner that is formed by overlapping blocks
curve. It is commonly found in Islamic of stone or brick that gradually project
architecture, especially in Spain and towards the center until they meet at the
North Africa. top.
Image

Arch type Shouldered Arch


Description A shouldered arch is an arch that has a flat section at the base of each side, creating
a rectangular shape.
Image

Q.58 Match the architectural projects in Group I with their corresponding architects in Group II.
(A) P-4, Q-5, R-2, S-3 Group I Group II
(B) P-4, Q-1, R-5, S-2 (P) Indian Institute of Management (1) Revathi Kamath
(C) P-2, Q-4, R-5, S-1 Bangalore
(Q) Osho International Meditation (2) Brinda Somaya
(D) P-3, Q-5, R-1, S-2
Resort, Pune
(R) Nalanda International School, (3) Roger Anger
Vadodara
(S) Matrimandir, Auroville (4) B. V. Doshi
(5) Hafeez Contractor
Ans:- (A)
Explanation:
The Indian Institute of Management Bangalore is a prestigious business school in India, designed by
B. V. Doshi, a renowned architect and urban planner who won the Pritzker Prize in 2018. The campus
features a blend of modern and traditional elements, such as courtyards, terraces, and red brick walls.
The Osho International Meditation Resort in Pune, India, was designed by renowned Indian architect
Hafeez Contractor. He is known for his distinctive modernist style, often incorporating elements of
traditional Indian architecture into his designs.
Hemant Vilas Parulekar 75

The architect behind the beautiful Nalanda International School in Vadodara is the renowned Brinda
Somaya, a name synonymous with sustainable and culturally sensitive design in India.
She is also the founder and principal architect of Somaya & Kalappa Consultants, a Mumbai-based
architectural firm that specializes in conservation, restoration, and sustainable design. She has won
several awards and honors for her work, such as the UNESCO Asia-Pacific Heritage award, the Leading
European Architects Forum award, and the Pritzker Architecture Prize. She is also a founder trustee of
the HECAR Foundation, which promotes the works of women architects in South Asia.
The Matrimandir is a spherical temple and the focal point of Auroville, an experimental township in
Tamil Nadu. The temple was designed by Roger Anger, a French architect and disciple of Sri Aurobindo
and The Mother, the founders of Auroville. The temple has a golden metallic exterior and a white
marble interior, symbolizing the sun and the soul.
Q.59 Match the structural joining systems in Group I with the corresponding materials for which they
are commonly used in Group II.
(A) P-4, Q-1, R-2, S-5 Group I Group II
(B) P-3, Q-5, R-1, S-2 (P) Welding (1) Glass
(C) P-2, Q-3, R-5, S-1 (Q) Spider Connector (2) Plastic
(R) Mortise and Tenon (3) Brick
(D) P-4, Q-1, R-5, S-3
(S) Mortar (4) Steel
(5) Timber

Ans:- (D)
Explanation:
Welding is a process of joining two pieces of metal by melting them together using high heat. It is
commonly used for steel structures, such as bridges, buildings, and pipelines.
Spider connector is a type of glass fitting that consists of a metal hub and several arms that attach to the
glass panels. It is commonly used for glass facades, canopies, and balustrades.
Mortise and tenon is a type of wood joint that consists of a hole (mortise) in one piece of wood and
a protruding tongue (tenon) in another piece of wood that fits into the hole. It is commonly used for
timber frames, furniture, and doors.
Mortar is a mixture of sand, water, and cement or lime that is used to bond bricks or stones together. It
is commonly used for brick walls, chimneys, and fireplaces.
Q.60 Match the Instruments in Group I with the corresponding climate parameters in Group II.
(A) P-3, Q-5, R-1, S-2 Group I Group II
(B) P-3, Q-4, R-5, S-2 (P) Pyranometer (1) Humidity
(C) P-5, Q-3, R-2, S-4 (Q) Disdrometer (2) Wind
(R)Hygrometer (3) Solar Radiation
(D) P-1, Q-2, R-3, S-5
(S)Anemometer (4) Pressure
(5) Precipitation
76 GATE Architecture and Planning: Comprehensive Question Bank

Ans:- (A)
Explanation:
A pyranometer is an instrument that measures solar radiation on a planar surface. It has a thermopile
sensor that detects the difference in temperature between a black surface that absorbs radiation and a
reference surface that does not. It also has a glass dome that limits the spectral response and protects
the sensor from environmental factors.
A disdrometer is an instrument that measures the size and velocity of raindrops. It uses a laser beam
or a piezoelectric sensor to count and classify the raindrops that fall through a sampling area. It can
provide information about the type and intensity of precipitation.
A hygrometer is an instrument that measures humidity, which is the amount of water vapor in the
air. There are different types of hygrometers, such as hair hygrometers, psychrometers, and electronic
hygrometers, that use different methods to measure the change in moisture content or relative humidity.
An anemometer is an instrument that measures wind speed and wind pressure. It usually has three
cups or propellers that spin around a vertical axis when the wind blows. The faster the cups spin, the
higher the wind speed. Anemometers are important tools for meteorologists and physicists who study
the weather and the air movement.
Q.61 In traditional Indian temple architecture, which of the following statement(s) is/are true?
(A) Jagamohana refers to a dancing hall
(B) Gopuram refers to an entrance tower
(C) Char-chala refers to a roof composed of four triangular segments
(D) Vimana refers to the structure over the Garbhagriha
Ans:- B, C, D
Explanation:
(A) Jagamohana: A jagamohana is a porch or hall that is attached to the main shrine of a Hindu temple.
It is usually the first structure that a visitor encounters after entering the temple compound. It serves
as a place for devotees to gather, worship, and witness rituals. The term jagamohana literally means
“delighting the world”.
(B) Gopuram: A gopuram is a monumental tower that marks the entrance of a Hindu temple, especially
in South India. It is usually decorated with sculptures, paintings, and carvings depicting various gods,
goddesses, and mythological scenes. The term gopuram means “cow’s
horn”, as it resembles the shape of a cow’s horn.
(C).Char-chala: A char-chala is a type of roof that is composed of four triangular segments that meet
at a point. It is a characteristic feature of Bengali temple architecture, especially in West Bengal and
Bangladesh. The term char-chala means “four slopes”.
(D) Vimana: A vimana is a structure that covers the garbhagriha or the sanctum sanctorum of a Hindu
temple. It is usually shaped like a pyramid, dome, or spire, and it represents the mythical abode of the
deity. The term vimana means “flying palace” or “vehicle”.
Q.62 Which of the following factors impact Daylight Autonomy of a built space?
(A) Orientation of building (B) Glare caused by daylight
(C) Latitude and longitude of building location (D) Fenestration size
Hemant Vilas Parulekar 77

Ans:- A, C, D
Explanation:
The factors that impact Daylight Autonomy of a built space are:
(A) Orientation of building: The orientation of the building affects the amount and direction of sunlight
that enters the space. Different orientations have different advantages and disadvantages depending on
the climate and the season. For example, south-facing windows can provide more daylight and passive
heating in winter, but may cause overheating and glare in summer.
(B) Glare caused by daylight: Glare is the sensation of discomfort or reduced visibility caused by
excessive brightness or contrast in the visual field. Glare can reduce the quality and usability of daylight,
and may require shading devices or artificial lighting to mitigate its effects. Glare can be caused by
direct sunlight, reflected sunlight, or bright sky.
(C)Latitude and longitude of building location: The latitude and longitude of the building location
affect the angle and duration of sunlight that reaches the space. Higher latitudes have lower sun angles
and shorter daylight hours in winter, and higher sun angles and longer daylight hours in summer.
Longitude affects the time zone and the solar time of the location, which may differ from the standard
time.
D) Fenestration size: Fenestration is the arrangement, design, and size of windows and other openings
in a building. Fenestration size affects the amount and distribution of daylight that enters the space.
Larger fenestration sizes can provide more daylight and views, but may also cause more heat loss or
gain, glare, and noise. Fenestration size should be balanced with other factors, such as orientation,
shading, insulation, and ventilation.
Q.63 For the beam shown in the following
Figure, assuming a sagging moment
(generating tensile stresses at the bottom
fibre) as positive and a hogging moment
(generating tensile stresses at the top fibre)
as negative, the bending moment
(in kN.m, rounded off to one decimal place)
at section X-X is ___________.
Ans: - 20
Explanation:
Rw + Ry = (4 x 20 + 120) = 200 kN
Taking moment at w =0
(20 x 4 x 2) – (Ry x 4) + (120 x 5) = 0
160 – 4Ry + 600 = 0
Ry = 190kN
Thus, Rw = 10 kN
Thus, bending moment at X-X = (Rw x 2) – (20 x 2 x 1) = 20 -40 = -20 kN.m
78 GATE Architecture and Planning: Comprehensive Question Bank

Q.64 The acoustical absorption of a wall panel in each octave band is tabulated below.
The Noise Reduction Coefficient of the wall panel (rounded off to two decimal places) is
___________.
63 125 250 500 1000 2000 4000 8000 16000
Hz Hz Hz Hz Hz Hz Hz Hz Hz
0.1 0.2 0.5 0.5 0.7 0.8 0.8 0.9 0.9
Ans:- 0.60 to 0.65
Explanation:
Noise Reduction Coefficient = arithmetic average of sound absorption coefficient for
Frequencies given, Thus, here Noise Reduction Coefficient = (0.1 + 0.2 + 0.5 + 0.5 + 0.7 + 0.8 + 0.8 +
0.9 + 0.9)/9 = 5.4/9 = 0.60
Q.65 A room is maintained at a wet bulb temperature of 25 oC, globe temperature of 30 oC, and air
velocity of 0.5 m/s. The decrease in Tropical Summer Index when the air velocity is increased to 3
m/s (in oC, rounded off to two decimal places) is ___________.
Ans: - 2.20 to to -1.50 or 1.50 to 2.20
Explanation:
Please Refer GATE-2018, Q. 11, for information related to definition and formula of ‘Tropical
Summer Index (TSI)’
TSI = 1/3 tw + 3/4 tg - 2V 1/2
Where, TSI = Tropical Summer Index, tw = Wet bulb Temperature (0C), tg = Globe temperature (0C), V
1/2
= Square root of air velocity (m/s)
Thus, in case 1st, TSI1 = (1/3 x 25) + (3/4 x 30) - 2(0.5)1/2
= 8.34 +22.5 - 1.41 = 29.43
In case 2nd, the air velocity increased to 3m/s
Thus, TSI2 = (8.34 = 22.5) - 2(3)1/2 = 30.84-3.46 = 27.38
Thus, decrease in Tropical Summer Index = 29.43 - 27.38 = 2.05

PART B2: FOR Planning CANDIDATES ONLY


Q.66 – Q.72 Carry ONE mark Each
Q.66 Which of the following is the National Electronic Toll Collection System implemented by the
National Payment Corporation of India?
(A) e-Pass
(B) E-ZPass
(C) HashTag
(D) FASTag
Hemant Vilas Parulekar 79

Ans:- (D)
Explanation:
The National Electronic Toll Collection System implemented by the National Payment Corporation
of India is (D) FASTag. FASTag is a device that employs Radio Frequency Identification (RFID)
technology for making toll payments directly while the vehicle is in motion. FASTag is linked to a
prepaid or savings/current account and the toll fare is deducted from the linked account. FASTag offers
the convenience of cashless payment and reduces traffic congestion at toll plazas.
Q.67 The shaded area in the following demand-supply
graph is known as ___________.
(A) Consumer Surplus ‘
(B) Consumer Deficit
(C) Producer Surplus
(D) Producer Deficit

Ans:- (A)
Explanation:
The shaded area shows, that ‘price’ of the commodity decreases if the ‘supply’ increases beyond the
consumer demand (I.e., consumer surplus).
Q.68 Identify the following traffic interchange.
Directional
Trumpet
Clover-Leaf
(D) Diamond

Ans:- (C)
Explanation: Please refer (Q. 18) GATE-2017 for more information
Q.69 Which of the following is/are Value Capture Method(s)?
(A) Building construction fees
(B) Fees for changing agricultural to non-agricultural land use
(C) User charge
(D) Premium on additional FSI/FAR
80 GATE Architecture and Planning: Comprehensive Question Bank

Ans:- B, D
Explanation:
Value capture is a type of public financing that recovers some or all of the value that public infrastructure
generates for private landowners. It can be used to offset the costs of providing and maintaining public
services and amenities, such as roads, parks, schools, and transit.
There are different methods of value capture that can be applied depending on the context and the
objectives of the public sector. Some of the common methods are:
Land value tax: A tax levied on the unimproved value of land, which reflects the benefits of public
infrastructure and services. This tax encourages efficient land use and discourages land speculation.
Betterment levy: A one-time charge imposed on landowners who benefit from a specific public project,
such as a new road or a rezoning. This charge captures a portion of the increase in land value resulting
from the project.
Development charges: A fee charged to developers who build new projects that require additional
public infrastructure and services, such as water, sewer, and roads. This fee covers the costs of providing
and expanding the infrastructure and services4
Tax increment financing: A mechanism that uses the future increase in property tax revenue from
a designated area to finance public improvements within that area, such as urban renewal or transit
projects. This mechanism stimulates private investment and development in the area5
Joint development: A partnership between the public and private sectors to develop land owned or
acquired by the public sector, such as land near transit stations. The public sector provides the land and
the private sector provides the capital and expertise to build mixed-use projects that generate revenue
and ridership for the public sector.
Transfer of development rights: A scheme that allows landowners to sell or transfer their development
rights to other landowners who want to increase their development potential, such as building height or
density. The public sector regulates the transfer of development rights to preserve certain areas, such as
heritage or environmental sites, and to promote development in other areas, such as transit corridors.
Q.70 Which among the following is/are model(s) of Public-Private Partnership (PPP) used for
infrastructure projects?
(A) BOLD (B) BOLT (C) BOOT (D) BPOT
Ans:- B, C
Explanation:
Public-Private Partnership (PPP) is a form of collaboration between a government agency and a private-
sector company that can be used to finance, build, and operate projects, such as public transportation
networks, parks, and convention centres. There are different models of PPP that vary in the degree
of involvement and risk sharing of the public and private partners. Some of the commonly adopted
models of PPP are:
BOT (Build-Operate-Transfer): The private partner builds the project, operates it for a specified
period of time, and then transfers it to the public partner.
BOO (Build-Own-Operate): The private partner builds the project, owns it, and operates it indefinitely,
without transferring it to the public partner.
BOOT (Build-Own-Operate-Transfer): The private partner builds the project, owns it, operates it for
a specified period of time, and then transfers it to the public partner.
Hemant Vilas Parulekar 81

BLT (Build-Lease-Transfer): The private partner builds the project, leases it to the public partner for
a specified period of time, and then transfers it to the public partner.
BOLT (Build-Own-Lease-Transfer): The private partner builds the project, owns it, leases it to the
public partner for a specified period of time, and then transfers it to the public partner.
DBFO (Design-Build-Finance-Operate): The private partner designs, builds, finances, and operates
the project for a specified period of time, and then transfers it to the public partner.
LDO (Lease-Develop-Operate): The public partner leases the project to the private partner, who
develops and operates it for a specified period of time, and then transfers it to the public partner.
OMT (Operate-Maintain-Transfer): The public partner builds the project and transfers it to the
private partner, who operates and maintains it for a specified period of time, and then transfers it back
to the public partner.
Q.71 The measured spot speeds (in km/h) of 10 vehicles from a traffic stream are 45, 35, 25, 51, 45, 38,
61, 42, 47, and 49. The Time Mean Speed of the traffic stream (in km/h, rounded off to one decimal
place) is ___________.
Ans:- 43 to 44
Solution:
To find the time mean speed of the traffic stream, you need to sum all of the spot speeds and divide by
the number of spot speeds.
The sum of the spot speeds is 45 + 35 + 25 + 51 + 45 + 38 + 61 + 42 + 47 + 49 = 438. Dividing this sum
by the number of spot speeds, which is 10, gives a time mean speed of 438/10 = 43.8 km/h. Therefore,
the time mean speed of the traffic stream is 43.8 km/h.
Q.72 In a township, the price of each house was 25,00,000 (in Indian Rupees) last month.
The number of houses sold in a month (Q in thousands) is sensitive to the price of the house
(P in Indian Rupees) and establishes a relationship as Q = 6685 – 0.00158P. If the price of each
house increases by 20% in the current month, then the decrease in sale of the houses (in percentage,
rounded off to two decimal places) compared to last month will be ___________.
Ans:- 27.00 to 30.00
Explanation:
The question based on ‘Price Elasticity of Demand (PED)’was earlier asked in (Q.24)GATE-2019
Assume price of each house sold last month as (P1) 25,00,000
Thus, as per given relation between no. of houses sold = Q1 = 6685-0.00158P1
& price of each house, the number of houses sold last = 6685-0.00158 x 2,50,000
month (Q1)
= 6685-3950
Q1 = 2735
Assume the increased price (by 20%) in the current P2 = P1 + (20% of P1)
month as (P2) 30,00,000
Thus, the number of houses sold (Q2) will be Q2 = 6685 – 0.00158P2
= 6685 – (0.00158 x 30,00,000)
Q2 = 1945
82 GATE Architecture and Planning: Comprehensive Question Bank

Thus, net decrease in sale of houses will be Q1 – Q2 = 2735 – 1945 = 790


Thus, % decrease in sale of houses compared to last (790/2735) x 100 = 28.89
month will be
Q.73 Match the models in Group I with their corresponding applications in Group II
(A) P-2, Q-1, R-5, S-4 Group I Group II
(B) P-1, Q-5, R-2, S-3 (P) Logit model 1.Trip assignment
(C) P-2, Q-3, R-5, S-4 (Q) Greenshield model 2.Modal split
(R) Gravity model 3.Traffic flow
(D) P-5, Q-3, R-4, S-2
(S) Multiple regression model 4.Trip generation
5.Trip distribution
Ans:- (C)
Explanation:
(P) Logit model: A statistical model that estimates the probability of an event occurring based on a
given dataset of independent variables. It can be used for classification and predictive analytics, and it
produces a probability between 0 and 1.
(Q) Greenshield model: A traffic flow model that assumes a linear relationship between speed and
density of vehicles on a road. It can be used to estimate the free flow speed, the jam density, the maximum
flow, and the optimum speed and density of traffic.
(R) Gravity model: A model that predicts the volume of flows of goods, services, or people between two
or more locations based on the population size and distance of the locations. It can be used to analyse
trade, migration, transportation, and other types of movement between places.
(S) Multiple regression model: A statistical model that estimates the relationship between two or more
independent variables and one dependent variable. It can be used to test hypotheses, explain variation,
and make predictions based on the data.
Trip assignment: The process of determining the routes that travellers will take to reach their destinations
based on the travel demand and the network characteristics.
Modal split: The proportion of travellers using different modes of transportation, such as car, bus, bike,
or walk.
Traffic flow: The movement of vehicles on a road or a network of roads. It can be characterized by
variables such as speed, density, and flow.
Trip generation: The process of estimating the number of trips that will be originated or attracted by
different zones or land uses.
Trip distribution: The process of estimating the number of trips that will be made between different
zones or lands uses.
Hemant Vilas Parulekar 83

Q.74 Match the proponents in Group I with the corresponding theories in Group II.
Group I Group II
P-2, Q-4, R-3, S-5 James Q Wilson and Creative Class
(B) P-4, Q-2, R-5, S-1 George K. Kelling
(C) P-5, Q-4, R-2, S-1 Sherry Arnstein Right to City
(R) Henry Lefebvre Drive-in Culture
(D) P-3, Q-5, R-2, S-4
Richard Florida Ladder of Citizen Participation
Broken Window
Ans:- (C)
Explanation:
Theory Description
Creative Class The term was coined by American urban studies theorist Richard Florida, who
argued that the creative class is a key driving force for economic development of
post-industrial societies. Florida identified two main groups within the creative
class: the super-creative core, which includes occupations that fully engage in
the creative process, such as computer programmers, scientists, writers, and
musicians; and the creative professionals, which include occupations that draw on
complex bodies of knowledge to solve specific problems, such as lawyers, doctors,
teachers, and engineers.
Florida also proposed that the creative class is attracted to places that have the
“three T’s” of economic growth: talent, technology, and tolerance. Talent refers
to the availability and diversity of skilled workers; technology refers to the level
of innovation and infrastructure; and tolerance refers to the openness and
acceptance of different cultures, lifestyles, and identities. Florida suggested that
cities and regions that have high levels of the three T’s can foster a creative and
entrepreneurial environment that stimulates economic and social progress.
Florida’s theory of the creative class has been influential and controversial in the
fields of urban planning, economic development, and cultural studies. Some
critics have challenged his empirical evidence, his causal claims, his normative
assumptions, and his policy implications. Some supporters have applied and
extended his ideas to different contexts, such as countries, industries, and
organizations
Right to City Right to the city is a concept and slogan that emphasizes the need for inclusivity,
accessibility, and democracy in urban spaces. The idea was first articulated by
French philosopher Henri Lefebvre in his 1968 book Le Droit à la Ville, in which
he argued that urban space should not be solely controlled by market forces, such
as commodification and capitalism but should be shaped and governed by the
citizens who inhabit it. The concept of the right to the city has been taken up by
a variety of social movements and urban activists around the world, who use it as
a rallying cry for greater social justice and democracy in the urban environment.
The right to the city can encompass a variety of demands, including demands for
government subsidized housing, access to public space, participation in urban
governance, and laws against displacement and gentrification, all of which aim to
address spatial inequalities in urban areas.
84 GATE Architecture and Planning: Comprehensive Question Bank

Ladder of The Ladder of Citizen Participation is a model proposed by Sherry Arnstein in


Citizen 1969, which describes the different levels of involvement and power that citizens
Participation can have in decision-making processes that affect their lives and communities.
The model uses the metaphor of a ladder with eight rungs, each representing a
degree of participation from non-participation to citizen power. The rungs are:
Manipulation: The lowest level of non-participation, where citizens are used as
puppets by power-holders to legitimize their decisions and actions.
Therapy: Another form of non-participation, where citizens are treated as passive
and ignorant patients who need to be cured by power-holders through education
or counselling.
Informing: A basic level of participation, where citizens are given access to
information by power-holders, but have no channel for feedback or influence.
Broken Window James Q. Wilson and George Kelling first introduced the Broken Window Theory
in 1982. It was based on their belief that signs of disorder, such as broken windows,
act as a signal that no one cares about the environment. This can lead to further
criminal activities like vandalism and theft.
By implementing policies and procedures that create a clean and orderly
environment, you can send the message that your business values order and
respect. This can help deter criminal activity and create a sense of safety.
It is important to remember that the Broken Window Theory is more than just
about physical disorder. It can also be applied to non-physical aspects of your
business, such as customer service and workplace policies.
Q.75 Match the Artists/Scientists in Group I with their corresponding contributions in Group II.
P-4, Q-3, R-1,S-2 Group I Group II
P-4, Q-1, R-5, S-2 Robert Park and Louis Wirth Poverty Map
P-5, Q-3, R-1, S-4 (Q)Jacob August Riis Cholera Map
(R)Charles Booth Tenement Shelter Photography
(D) P-4, Q-3, R-5, S-1
John Snow Urban Ethnography
Underground Sewerage Systems
Ans :- (A)
Explanation:
To answer this question, we need to know the contributions of the artists/scientists in Group I to the
fields of sociology, public health, and social reform. Here is a brief summary of their achievements:
(P) Robert Park and Louis Wirth were American sociologists who were part of the Chicago School of
Sociology. They developed a theoretical and empirical approach to the study of urban life, focusing on
issues such as immigration, assimilation, social ecology, and collective behaviour. They are considered
pioneers of urban sociology and urban ethnography.
(Q) Jacob August Riis was a Danish-American journalist and social reformer who used photography
and journalism to expose the harsh conditions of the poor and the immigrants in New York City’s
tenements. He published his famous book How the Other Half Lives in 1890, which shocked the public
and inspired social change. He is regarded as one of the first muckrakers and social documentary
photographers.
Hemant Vilas Parulekar 85

(R) Charles Booth was a British shipowner and social researcher who conducted a massive survey
of the life and labour of the people in London from 1886 to 1903. He collected and analysed data
on poverty, industry, religion, and other aspects of urban society. He produced a series of maps that
showed the distribution of poverty and wealth in different areas of the city. He also advocated for social
reforms such as old-age pensions and improved sanitation. He is considered one of the founders of
social administration and social policy.
(S) John Snow was a British physician and epidemiologist who is regarded as the father of modern
epidemiology and early germ theory. He is best known for his investigation of the cholera outbreak in
London in 1854, where he traced the source of the infection to a contaminated water pump on Broad
Street. He also conducted a natural experiment that compared the mortality rates of different water
suppliers in the city. He also made significant contributions to the development of anaesthesia and
medical hygiene.
Q.76 In the conceptual diagram of the city given below, P, Q, R, and S refer to urban patterns. Among the
choices given below, the correct association is

(A) P-Satellite town, Q-Urban fringe, R-TOD, S-Central Business District


(B) P-Central Business District, Q-Satellite town, R-TOD, S-Urban fringe
(C) P-Urban fringe, Q-TOD, R-Satellite town, S-Central Business District
(D) P-Satellite town, Q-Central Business District, R-TOD, S-Urban fringe
Ans:- (A)
Explanation:
Please refer GATE-2018, (Q. 36) (TOD) for more information.
A satellite town is a smaller town that is located near a larger city and is dependent on it for economic
and social activities.
An urban fringe is the outer edge of a city, where urban and rural land uses mix and often conflict.
A TOD (transit-oriented development) is a type of urban development that maximizes the amount of
residential, business, and leisure space within walking distance of public transport3.
A central business district (CBD) is the commercial and business center of a city, where land values are
high and building densities are greatest.
86 GATE Architecture and Planning: Comprehensive Question Bank

Based on these definitions, we can compare the choices with the diagram and see which one matches
the best. The diagram shows four zones (P, Q, R, and S) that are connected by a circular rail line. The
zones have different sizes, shapes, and colors, which indicate their different characteristics.
Option (A) is incorrect, because P is not a satellite town, but a large and dense zone that resembles a
CBD. Q is not an urban fringe, but a smaller and less dense zone that resembles a satellite town. R is not
a TOD, but a zone that has no rail access and is isolated from the other zones. S is not a CBD, but a zone
that has rail access and is close to the other zones, which resembles a TOD.
Q.77 Which among the following is/are the component(s) of the assimilative carrying capacity of urban
environment?
(A) Air (B) Water (C) Economy (D) Soil
Ans:- A, B, D.
Explanation:
Assimilative carrying capacity refers to the ability of the environment or a portion of the environment
to carry waste material without adverse effects on the environment or on users of its resources1. It
is a concept that involves both natural and human factors, such as resources, ecosystems, pollution,
population, and development.
Based on this definition, the components of the assimilative carrying capacity of urban environment
are (A) Air, (B) Water, and (D) Soil. These are the natural resources that can absorb and degrade the
waste generated by urban activities, such as transportation, industry, and consumption. However, the
assimilative capacity of these resources is limited by the quality and quantity of the resources, as well as
the type and amount of the waste.
(C) Economy is not a component of the assimilative carrying capacity of urban environment, because
it is not a natural resource that can carry waste material. Rather, it is a human factor that affects the
demand and supply of resources, and the level and pattern of development in urban areas.

Q.78 In the transportation network given below, P, Q, R, S, T,


and U are the nodes and values mentioned on the links
denote time in minutes. Which of the following options
represent the minimum spanning tree? (MSQ)
(A) PQ, QR, QT, TS, SU (B) PR, QR, RT, TU, SU
(C) PQ, QR, RT, TS, SU (D) PQ, QR, RS, ST, TU
Ans:- A, C
Explanation:
A minimum spanning tree (MST) is a subset of the edges of a connected, edge-weighted undirected
graph that connects all the vertices together, without any cycles and with the minimum possible total
edge weight graph /transportation network can have multiple spanning trees. To find the MST of a
given graph, we can use algorithms such as Kruskal’s or Prim’s2.
Using Kruskal’s algorithm, we can follow these steps:
Sort all the edges by their weights in ascending order.
Hemant Vilas Parulekar 87

Pick the smallest edge that does not form a cycle with the edges already in the MST. Repeat until there
are n - 1 edges in the MST, where n is the number of vertices.
Using Prim’s algorithm, we can follow these steps:
Start with any vertex and mark it as visited.
Pick the smallest edge that connects a visited vertex to an unvisited vertex. Add it to the MST and mark
the unvisited vertex as visited. Repeat until all the vertices are visited.
Using either algorithm, we can find that the MST of the given graph is option The weight of a spanning
tree is sum of weights given to each edge (‘node’ in case of transportation network) of the spanning tree.
A minimum spanning tree has (V-1) edges (nodes), where ‘V’ is the number of vertices I the given
graph/network. In the given problem, the minimum spanning tree has (6-1)b’5’ nodes, From the
definition, the weight of spanning tree mentioned in options A, B, C, & D can be calculated as shown
in the adjacent table.
It can be observed that, option A & C shows minimum spanning tree with total weight of 12 minutes.
Option Nodes Total Weight (time)
A PQ QR QT TS SU 12
2 2 3 3 2
B PR QR RT TU SU 14
2 2 3 4 2
C PQ QR RT TS SU 12
2 2 3 3 2
D PQ QR RS ST TU 17
2 2 6 3 4
Q.79 A vehicle count survey (in Passenger Car Unit) is conducted on a mid-block section of a road at
regular intervals of 15 minutes from 8:00 AM to 10:00 AM. Based on the data given in Table below,
the Peak Hour Factor (rounded off to two decimal places) for the given survey duration is ______
_____.
Time Interval Passenger Car Unit
8:00 AM – 8:15 AM 212
8:15 AM – 8:30 AM 248
8:30 AM – 8:45 AM 272
8:45 AM – 9:00 AM 315
9:00 AM – 9:15 AM 337
9:15 AM – 9:30 AM 405
9:30 AM – 9:45 AM 320
9:45 AM – 10:00 AM 267
Ans:- 0.83 to 0.87
Explanation:
Peak hour volume is a term used in traffic engineering to measure the volume of traffic that uses a road
segment, an intersection, or a lane during the hour of the day that observes the highest traffic volumes.
It is usually expressed in terms of passenger car units (pcu), which is a unit that converts different types
of vehicles into equivalent numbers of passenger cars based on their size and impact on traffic flow. It
can be identified as between 8.45 AM to 9.45 AM
88 GATE Architecture and Planning: Comprehensive Question Bank

Thus, Peak Traffic Volume (in PCU) during this hour = 315 + 337 + 405 + 320 =1377
Peak-Hour Factor (PHF): It is the average volume during the peak 60 minutes (hour) divided by four
times the average volume during the peak 15 minutes’ period.
Actual (Design) flow rate: It is calculated by dividing the peak hour volume by Peak Hour Factor (PHF)
or by multiplying the peak 15-minute volume by four.
Thus, Actual design flow rate = 1377/0.85
= 1620 PCU/h
Or, 405 x 4 = 1620 PCU/h
Time Interval Passenger Car Unit
8:00 AM – 8:15 AM 212
8:15 AM – 8:30 AM 248
8:30 AM – 8:45 AM 272
8:45 AM – 9:00 AM 315
9:00 AM – 9:15 AM 337
9:15 AM – 9:30 AM 405
9:30 AM – 9:45 AM 320
9:45 AM – 10:00 AM 267
Q.80 A land owner has shown interest in a Town Planning Scheme. Based on the details of the scheme
given in the following Table, the estimated Net Benefit to the land owner after land development
(in Indian Rupees, in integer) is ___________. in the following Table, the estimated Net Benefit to
the land owner after land development (in Indian Rupees, in integer) is ___________.
Original plot size 500 Sq. m
Original land value 1200 Indian Rupees per Sq.m
Plot deduction for development 40%
Developed land value 2800 Indian Rupees per Sq.m
Total betterment cost to be paid by the land owner 50% of the increased total land value
Ans:- 118000 to 122000
Solution:
Given, Original plot size (underdeveloped land) = 500 sq.m.
Given, Original land value (Undeveloped land) = 1200 INR per sq.m.
Thus, original value of total undeveloped plot = 1200 x 500 = Rs. 6,00,000
Given, Plot deduction for development = 40% of original plot size
Thus, net deduction of plot = 0.4 x 500 = 200 sq.m.
Thus, after deduction, net area of developed plot = 300 sq.m.
Given, Developed land value per sq.m. = 2800 INR per sq.m.
Thus, total land value = 2800 x 300 = Rs.8,40,000
Thus, net increase in land value after development = 8,40,000 - 6,00,000 = Rs. 2,40,000
Given Total betterment cost to be paid by the land owner = 0.5 x 2,40,000 = Rs. 1,20,000
Thus, Net benefit to land owner = 2,40,000 - 1,20,000 = Rs. 1,20,000.
Hemant Vilas Parulekar 89

Q.81 The year-wise cash flows (in Indian Rupees) of a construction project are given in the following
Table. If the annual discount rate for the project is assumed to be 12%, the Net Present Value (in
Indian Rupees, rounded off to two decimal places) for the project will be ___________.
Year Annual Cash Outflow Annual Cash Inflow
0 5,00,000 0
1 0 0
2 0 0
3 50,000 1,80,000
4 50,000 2,20,000
5 50,000 2,90,000
6 0 3,30,000
Ans: 3800 to 4200
Solution:
Net Present Value = Present Value of Cash Inflow - Present value of Cash Outflow…………..(A)
Discounting Factor (Dn) = 1 / (1+r)n
Where, r = Annual Discount rate
n = number of years ahead
The discounting factor for year 0 will always be ‘1’
Thus, by using this the discounting factor for year ‘1’ can be calculated as follows:
Dn = 1/(1+r)n = 1/(1+0.12)1 = 0.8929
Likewise, the discounting factor factor for year 2 to 6 can be derived as shown adjacent
Year Discounting Factor
0 1
1 0.8929
2 0.7971
3 0.7117
4 0.6355
5 0.5674
6 0.5066
As, Present value of annual cash outflow = Annual cash outflow x Discounting factor & present
value of annual cash inflow = annual cash inflow x discounting factor
Thus, the present value of annual cash outflow and inflow can be calculated as follows:
Year Annual Cash Present Value Annual Cash Present Value (Inflow)
Outflow (Outlow) Inflow
0 5,00,000 5,00,000 0 0
1 0 0 0 0
2 0 0 0 0
90 GATE Architecture and Planning: Comprehensive Question Bank

Year Annual Cash Present Value Annual Cash Present Value (Inflow)
Outflow (Outlow) Inflow
3 50,000 35,585 1,80,000 1,28,106
4 50,000 31,775 2,20,000 1,39,810
5 50,000 28,370 2,90,000 1,64,546
6 0 0 3,30,000 1,67,178
Total 5,95,730 Total 5,99,640
From the formula mentioned as (A), Net Present Value = 5,99,640 - 5,95,730 =3910

You might also like